Section 6 Flashcards

1
Q

A 16-year-old boy is being evaluated for his brain hemorrhage in the left temporal region. CT angiography revealed an arteriovenous malformation (AVM) with a 6 X 6 cm2 nidus supplied by multiple feeders from the middle and the posterior cerebral arteries and having venous drainage to the straight sinus. What would be the plan of management for the patient?
1. Surgical resection of the AVM
2. Radiation therapy
3. Embolization therapy
4. Supportive management

A

4. Supportive management

  • The resectability of the AVM is determined by the Spetzler Martin grading scale that takes into account the size of the nidus, the eloquence of the location of the AVM, and the pattern of venous drainage of the AVM.
  • The clinical scenario represents a high score in terms of the Spetzler margin grading scale with a nidus of greater than 6 cm, eloquent location as well as venous drainage to the confluence of sinuses.
  • A high score means a high risk for complications while planning surgical resection.
  • Radiation and embolization therapy is planned for lesions with borderline scores so as to downgrade the scores, thereby minimizing the risk associated with surgical resection of the AVMs.
How well did you know this?
1
Not at all
2
3
4
5
Perfectly
2
Q

A 16-year-old male presents in the emergency department after a motor vehicle accident from which he was thrown from the vehicle. Since the accident, the patient has not been able to move his legs and is anxious as a result. He has a past medical history of asthma. On examination, he has bilateral loss of motor function of the lower extremities, bilateral loss of pain and temperature, and urinary incontinence. Imaging demonstrates arterial injury at the level of T9 connecting to the anterior spinal artery. The injured artery arises from which of the following?
1. Descending aorta
2. Ascending aorta
3. Internal carotid artery
4. External carotid artery

A

1. Descending aorta

  • The artery of Adamkiewicz is found between T8 and L3.
  • The artery of Adamkiewicz arises from the descending aorta.
  • The Artery of Adamkiewicz is the major arterial supply of the anterior spinal artery.
  • Anterior cord syndrome (also called anterior spinal artery syndrome) most commonly occurs due to an interrupted supply of the anterior spinal artery or the Artery of Adamkiewicz (its major supplier), which has a less efficient supply compared to the 2 posterolateral spinal arteries.
How well did you know this?
1
Not at all
2
3
4
5
Perfectly
3
Q

A 35-year-old Hispanic female 22 weeks age of gestation presents to the clinic for a check-up. She has diabetes and does not have a regular prenatal check-up. Ultrasound reveals a small biparietal diameter, overlapping of the frontal bones, and curved cerebellum. Which of the following is the most common complication expected of her baby’s condition after she gives birth?
1. Seizures
2. Hydrocephalus
3. Pulmonary embolus
4. Acute renal failure

A

4. Acute renal failure

  • Spina bifida is a congenital anomaly that arises from incomplete development of the neural tube.
  • Women should undergo routine screening to identify neural tube defects early and help with therapeutic intervention and counseling. Initial screening is done with serum AFP, but in cases of high suspicion, amniocentesis can be pursued for confirmation. However, given the risk of amniocentesis and the accuracy of ultrasound, the latter has become the gold standard for diagnosis in-utero.
  • The most common complication is acute renal failure/urosepsis which is secondary to ureteral reflux caused by the neurogenic bladder.
  • Most deaths beyond age 5 are attributed to seizures, pulmonary emboli, hydrocephalus, and acute renal failure/sepsis.
How well did you know this?
1
Not at all
2
3
4
5
Perfectly
4
Q

A 35-year-old male is seen in the emergency department. He was involved in a high-speed car crash and was an unrestrained passenger. He suffered minor bruises and complains of back pain. Examination reveals bruising on his arm and face. There is localized tenderness over the lower back on deep palpation. He complains of severe pain on back extension. The examination of lower limbs reveals no sensory or motor deficits. A lateral X-ray of the lower spine shows an anterior translation of the fifth lumbar vertebrae over the sacral vertebrae. The displacement is measured to be approximately 75%, and there is a fracture of pars interarticularis. The rest of the spine is normal. What would be appropriate management in this patient?
1. Thoracolumbosacral orthosis
2. Bed rest and analgesia
3. Spinal fusion surgery
4. Abdominal muscle flexion exercises

A

3. Spinal fusion surgery

  • This patient has been involved in a high-speed moto vehicle collision. He complains of lower back pain but has no demonstratable focal neurological deficit. His X-ray spine reveals the presence of anterior fracture-dislocation of L5 vertebrae consistent with the diagnosis of traumatic lumbar spondylolisthesis.
  • Lumbar spondylolisthesis is the dislocation of the lumbar vertebrae over the vertebral body immediately below it. This patient has a high-grade dislocation and warrants surgical intervention. Spondylolisthesis with accompanying fracture of par interarticularis is considered unstable and required fixation.
  • Lumbar traumatic spondylolisthesis is a rare occurrence, and management decision remains controversial. Individuals with high-grade (greater than 50%) dislocation and those with progressive neurological symptoms are candidates for surgical intervention. Fusion in situ is the criterion standard in surgical intervention.
  • Conservative management can be considered in low-grade dislocations and involve rest, braces, and abdominal muscle strengthening exercises. Failure of conservative management or progression of symptoms requires surgical intervention.
How well did you know this?
1
Not at all
2
3
4
5
Perfectly
5
Q

A patient has an anterior abdominal incision after her transsphenoidal hypophysectomy. This most likely represents which of the following?
1. An incision to harvest a fat graft
2. An incision to harvest a fascia graft
3. An incision to insert a lumbar drain
4. An incision to remove a mole that the surgeon noticed for the first time during the surgery

A

1. An incision to harvest a fat graft

  • A fat or fascia graft may be required during transsphenoidal hypophysectomy to seal the hole in the dura.
  • Usually, a fat graft is obtained from the abdomen and a fascia graft from the thigh.
  • A lumbar drain is inserted posteriorly and does not require an actual incision.
  • A surgeon should not remove a mole unless the patient gives prior informed consent.
How well did you know this?
1
Not at all
2
3
4
5
Perfectly
6
Q

A 65-year-old presents with vague, dull pain in his hips for the past 2 months. The pain is often worse at night. Lately, he has been having headaches. He denies any trauma. Physical reveals bossing of the skull. The patient also appears to have an abnormal gait. Two weeks ago, he was admitted for heart failure. What is the most common neurological presenting feature of this patient’s disease?
1. Back pain
2. Urinary retention
3. Hearing loss
4. Diplopia

A

3. Hearing loss

  • The most common neurological problem in Paget disease is hearing loss.
  • During the bone remodeling, the cranial nerve VIII and cochlear function are disturbed.
  • Another neurological sign is pain or numbness along the course of a nerve if the spine is affected and the spinal nerve root (radiculopathy) is crushed.
  • Dizziness and headache can be due to the excessive growth of the skull bones.
How well did you know this?
1
Not at all
2
3
4
5
Perfectly
7
Q

A 46-year-old female presents to the emergency room with painful discoloration of her fingers. She has a history of similar intermittent episodes where her fingers turn white, then blue, followed by numbness and pain in the corresponding fingers. She was previously diagnosed with Raynaud’s syndrome, which has been refractory to medical management. The team plans for a stellate ganglion block for therapy. What is the location of the stellate ganglion in reference to its surgical landmarks?
1. Just inferior to the zygomatic arch
2. At the bifurcation of the carotid artery
3. Adjacent to the neck of the first rib and C7 transverse process
4. Posterior to the azygous vein

A

3. Adjacent to the neck of the first rib and C7 transverse process

  • This patient is suffering from Raynaud’s syndrome that is refractory to medical management. Stellate blocks are performed to decrease symptoms of hyperhidrosis or Raynaud. Blocks can also help treat reflex sympathetic dystrophy, currently known as complex regional pain syndrome.
  • The stellate ganglion is between the transverse process of the seventh cervical vertebra and the neck of the first rib. It is also known as the inferior cervical ganglion or the cervicothoracic ganglion.
  • An important surgical consideration is that stellate blocks can be complicated by Horner syndrome.
  • A successful stellate ganglion blockade is demonstrated by specific clinical signs, which include: miosis (pupil constriction), anhidrosis (lack of sweating), ptosis (drooping of the upper eyelid), and flushing of the extremities. These symptoms should resolve within 4 to 6 hours. Extremity temperatures may rise by 1 to 3 degrees, thereby indicating a successful block, which is the most widely employed indicator to determine the success of the stellate ganglion block.
How well did you know this?
1
Not at all
2
3
4
5
Perfectly
8
Q

A 65-year-old man presents to the emergency department complaining of drooling, slurred speech, and double vision that began 6 hours ago. On physical examination, the patient’s face appears asymmetrical, and when asked to smile, only half of his face can smile. The patient is also noted to have difficulty abducting his right eye during right horizontal gaze. A noncontrast CT of the brain reveals microvascular changes in the brainstem, and an MRI confirms an infarction in the caudal portion of the pons, lateral to the sulcus limitans. It is confirmed the patient suffered a stroke affecting the cranial nerve nucleus responsible for the horizontal conjugate gaze and abduction of the eye. Which of the following best explains why this patient is also suffering from ipsilateral facial nerve palsy and the inability to abduct his right eye?
1. The facial nucleus is located near the abducens nucleus
2. The facial nerve fascicle passes near the abducens nucleus
3. Fibers from the abducens nucleus innervate part of the contralateral face
4. The nucleus prepositus hypoglossi is located near the abducens nucleus

A

2. The facial nerve fascicle passes near the abducens nucleus

  • The abducens nucleus is responsible for conjugate horizontal gaze and abduction. The abducens nucleus is positioned beneath the floor of the fourth ventricle in the caudal portion of the pons and lateral to the sulcus limitans.
  • The abducens nucleus is separated from the fourth ventricle by the facial nerve genu, which curves over the dorsal and lateral surface of the abducens nucleus. An infarction of the abducens nucleus can also lead to damage to the facial nerve, causing ipsilateral facial paralysis.
  • In many cases of abducens nucleus damage, an ipsilateral facial nerve palsy will also be present, because the genu of the facial nerve fascicle passes around the abducens nucleus before exiting the brainstem.
  • The abducens nucleus does not innervate part of the ipsilateral face. However, the facial nerve genu runs near this nucleus, and damage to the abducens is commonly accompanied by damage to the facial nerve.
How well did you know this?
1
Not at all
2
3
4
5
Perfectly
9
Q

Which of the following conditions frequently are associated with basilar invagination?
1. Chiari malformation
2. Cervical level 2 (C2) fractures
3. Early onset dementia
4. Extremity paralysis

A

1. Chiari malformation

  • Chiari malformations are when the brain tissue extends down into the spinal canal, causing compression, whereas basilar invagination is when the bones of the cervical spine push up into the brain stem.
  • There is a wide array of presentations for basilar invagination with many of them being a direct result of the syndrome with which it is associated, such as a Chiari malformation which is associated with basilar malformations at a rate of 33% to 38%.
  • This crowding at the dens-atlas-clivus complex could obstruct cerebrospinal fluid flow leading to syringomyelia.
  • Both conditions are potentially life threatening, and advanced imaging studies such as an MRI are needed to evaluate the brain stem.
How well did you know this?
1
Not at all
2
3
4
5
Perfectly
10
Q

A 17-year-old male patient sustained a diffuse axonal injury within the brain stem following a traffic collision. Two weeks after the injury, he started developing episodes of tachypnea, tachycardia, diffuse sweating, and extensor posturing. There is no fever, and his complete blood count is unremarkable. A new computed tomography of the head did not reveal any progression in the brainstem lesions. What would be the next step in the management of the patient?
1. Fluid resuscitation
2. Beta-blockers
3. Broad-spectrum antibiotics
4. Paralyze the patient

A

2. Beta-blockers

  • A patient with diffuse axonal injury type 3 can develop paroxysmal autonomic instability with dystonia or dysautonomia.
  • The description of the clinical features of bouts of tachypnea, tachycardia, flushing, sweating, and posturing seen in the patient is classic for dysautonomia.
  • A beta-blocker such as propranolol has been shown to be beneficial in managing patients with features of dysautonomia.
  • Sepsis results in refractory hypotension and is associated with multiple organ failure. The patient has bouts of classic episodes of dysautonomia. There is no need for paralyzing the patient since there is no evidence of raised intracranial pressure as evident from the repeat CT brain.
How well did you know this?
1
Not at all
2
3
4
5
Perfectly
11
Q

A 3-month-old boy is being evaluated for a forehead bone ridge. There is no history of sleeping positional preference. Pregnancy was without complications, and there was no evidence of intrauterine constraint. A physical evaluation is being conducted to differentiate between a being metopic ridge and metopic craniosynostosis. Which of the following features is most consistent with metopic synostosis in this patient?
1. Hypertelorism
2. Hypotelorism
3. Bitemporal widening
4. Biparietal narrowing

A

2. Hypotelorism

  • The premature fusion of the metopic suture produces a narrow forehead, causing the eyes’ position to be closer than usual (hypotelorism) with associated bitemporal narrowing.
  • Trigonocephaly is the premature closure of the metopic suture forming a triangular forehead, with an obvious or subtle osseous ridge.
  • Metopic craniosynostosis should be distinguished from benign metopic ridge where hypotelorism and bitemporal narrowing are absent in the benign condition, and there are no other clinical features.
  • 3D computed tomographic scans can be used to differentiate them if a physical examination is not diagnostic.
How well did you know this?
1
Not at all
2
3
4
5
Perfectly
12
Q

A 28-year-old man is being evaluated for refractory epilepsy for the last 5 years. He is currently taking phenytoin, lamotrigine and gabapentin. Despite taking three medications, he had 2 episodes of seizure during last month. Surgery for control of seizure is planned. For precise excision of epileptic foci, surgery is planned as awake craniotomy. During the intraoperative mapping of epileptic foci, he starts having focal seizures in his right extremity. Which of the following is the next best step in the management of this patient?
1. Irrigation of the brain with iced saline
2. Abort the surgery and close craniotomy
3. Propofol
4. Fosphenytoin

A

1. Irrigation of the brain with iced saline

  • The first-line treatment of seizure during brain surgery is irrigation of the brain with sterile iced saline. Propofol bolus (10 to 20 mg IV) or midazolam (1 to 2 mg IV) should be administered to terminate the seizure if iced saline is not effective.
  • The incidence of seizure is 2-20% in awake craniotomy. A seizure occurs most commonly during stimulation for brain mapping. Many intraoperative seizures are focal, brief, and resolve spontaneously, whereas others are generalized.
  • Patients with a history of seizures and younger patients, especially with tumors of the frontal lobe, are more prone to seizures.
  • Intraoperative seizures are associated with a higher incidence of transient postoperative motor deterioration and protracted length of hospital stay.
How well did you know this?
1
Not at all
2
3
4
5
Perfectly
13
Q

A 43-year-old man presents to the clinic for a follow-up 2 weeks after an evaluation for a postural headache with associated diplopia and vertigo. A disease process at which of the following locations is most likely responsible for this patient’s headache?
1. Foramen of Lushka
2. Third ventricle
3. Spinal cord
4. Brainstem

A

3. Spinal cord

  • This patient has symptoms consistent with spontaneous intracranial hypotension, which is believed to most commonly occur from CSF leak.
  • The majority of CSF leaks are localized within the spine, most commonly within the thoracic or cervicothoracic junction.
  • Rarely, CSF leaks can occur at the skull base, although it is controversial whether this anatomical region of CSF leak relates to cases of spontaneous intracranial hypotension and orthostatic headache.
  • Studies have shown that patients presenting with a postural headache with suspicion of intracranial hypotension should be presumed to have a spinal source for CSF leak even if there is already evidence of skull base CSF leak.
How well did you know this?
1
Not at all
2
3
4
5
Perfectly
14
Q

A 42-year-old male with no significant past medical history presents after falling down a ladder. On physical examination, he appears drowsy but responds to commands and appears to have decreased sensation and strength in the left upper and lower extremity. A computed tomography scan of the head reveals a right-sided subdural hematoma. The patient is admitted for further observation and frequent neurologic assessments. Overnight, the patient becomes unresponsive and is now noted to have anisocoria with a fixed dilated pupil on the right. Which of the following may have contributed to his neurologic decline?
1. An influx of sodium and potassium
2. An efflux of calcium
3. An influx of sodium and calcium and efflux of potassium
4. Uptake of glucose and potassium

A

3. An influx of sodium and calcium and efflux of potassium

  • This patient shows signs of herniation with a fixed dilated pupil and a decline in mental status. This can occur due to the expansion of hematoma and vasogenic edema.
  • There is disruption of the blood-brain barrier, activation of inflammatory cells, and death of glial cells and neurons in traumatic brain injury. This is mediated by many processes, including uptake of sodium and calcium and the efflux of potassium.
  • Neurotransmitters associated with calcium channels in traumatic brain injury include histamine, glutamate, serotonin, and acetylcholine.
  • A number of cytokines cause calcium influx.
How well did you know this?
1
Not at all
2
3
4
5
Perfectly
15
Q

A 45-year-old woman presents to the provider complaining of high fever, left eye chemosis, and headache for the past three days. Past medical history is significant for a history of diabetes mellitus for the past ten years. Vital signs reveal a blood pressure of 120/80 mmHg, a pulse rate of 79 beats per minute, a respiratory rate of 14/min., and a temperature of 39 degrees Celsius. On physical examination, she has bilateral proptosis, and palsy of the sixth cranial nerve on the right side. Which of the following antibiotics should be included in the initial treatment of her underlying condition?
1. Vancomycin and cefotaxime
2. Cefotaxime and metronidazole
3. Vancomycin and metronidazole
4. Vancomycin, cefotaxime, and metronidazole

A

4. Vancomycin, cefotaxime, and metronidazole

  • An infected cavernous sinus thrombosis is a medical emergency. Even with early antibiotic therapy, the mortality rate is approximately 20 percent.
  • There are numerous potential sources of infection; therefore, the initial therapy must be broad-spectrum until definitive determination of the causative pathogen.
  • Recent research shows that anticoagulation is beneficial in one study, but thrombolytics have not yet been studied.
  • The dangerous area of the face consists of the triangular area from the corners of the mouth to the nasal bridge, including the lower part of the nose and maxilla. Venous drainage from this area drains directly into the cavernous sinus. Therefore, any infection involving the dangerous area of the face is a potential cause of cavernous sinus thrombosis.
How well did you know this?
1
Not at all
2
3
4
5
Perfectly
16
Q

A 65-year-old patient is rushed to the emergency department following sudden unresponsiveness. His Glasgow coma scale at presentation was only 9/15. While evaluating the patient, the treating provider found his eyeballs’ persistent fast conjugate downward movement followed by a slow return to the midpoint. Based on this clinical finding, what is the most likely localization of the lesion in the patient?
1. Midbrain
2. Pons
3. Medulla
4. Thalamus

A

2. Pons

  • The conjugate downward movement of the eyeballs followed by a slow return to the midpoint is characteristic of an ocular bobbing.
  • Ocular bobbing results due to the involvement of the para- pontine reticular formation that mediates horizontal gaze. The bobbing occurs due to the unopposed action of the interstitial nucleus of Cajal in the midbrain, governing vertical movements of the eyeballs.
  • Intrinsic pontine lesions can classically present with ocular bobbing, which has a high localizing value.
  • Lesions in the midbrain present with upward gaze palsy. The lesions in the thalamus sometimes show the eye peering towards the tip of the nose.
How well did you know this?
1
Not at all
2
3
4
5
Perfectly
17
Q

A 70-year-old female presents with persistent neck pain and worsening radicular pain down her left arm despite conservative treatment. Examination reveals diminished left triceps reflex, diminished sensation on the left middle finger, and left triceps weakness. Further evaluation with an MRI of the cervical spine showed multilevel degenerative changes, including a large posterolateral disc protrusion at C6-C7, causing severe left intervertebral foraminal stenosis. What is the definitive treatment for this patient’s pathology?
1. Physical therapy
2. Operative decompression
3. Non-steroidal anti-inflammatory drugs
4. Epidural steroid injections

A

2. Operative decompression

  • The patient is likely experiencing a C7 nerve root compression in the setting of cervical spondylosis. Initial treatment typically consists of conservative management, including physical therapy, soft cervical collar, traction, nonsteroidal anti- inflammatory drugs (NSAIDs), pain medications, epidural steroid injections, and medial branch blocks.
  • Surgical decompression is the only definitive treatment for progressively worsening radicular pain and symptoms resulting from a nerve root compression in the setting of a disc herniation.
  • Indications for surgery include progressive neurologic deficits, documented compression of a cervical nerve root and spinal cord, or intractable pain.
  • Surgical intervention is generally not indicated for axial neck pain alone and is typically reserved for individuals experiencing radicular and myelopathic symptoms.
How well did you know this?
1
Not at all
2
3
4
5
Perfectly
18
Q

What is the optimal treatment for a chordoma?
1. Chemotherapy
2. Radiation therapy
3. Surgery and chemotherapy
4. Surgery and radiation therapy

A

4. Surgery and radiation therapy

  • Chordomas are slow-growing tumors with a high local recurrence rate thus necessitating post-surgical radiation therapy.
  • The slow-growing nature of chordomas makes them relatively resistant to radiation therapy requiring high-dose conformal radiation therapy such as proton beam therapy for post- resection treatment.
  • Chemotherapy is uncommonly used for the treatment of chordomas as they are resistant to most current conventional chemotherapeutic agents.
  • Complete en bloc resection of a chordoma with clean margins can increase 5-year survival to 65% from 50% 5-year survival for resection with positive margins.
How well did you know this?
1
Not at all
2
3
4
5
Perfectly
19
Q

A 30-year-old male presents with left-sided hemiparesis. He is drowsy but responds to commands. His CT scan of the brain reveals a right frontal hemorrhage. MRI scan shows a combination of small ischemic and hemorrhagic lesions. What is the probable etiology?
1. Cerebral amyloid angiopathy
2. Hemorrhagic infarction
3. Cerebral vasculitis
4. Anticoagulation induced hemorrhage

A

3. Cerebral vasculitis

  • A combination of small ischemic and hemorrhagic lesions on MRI is the feature of cerebral vasculitis.
  • The patient’s young age also suggests vasculitis, compared to the other etiologies.
  • The usual causes of spontaneous subarachnoid hemorrhage are ruptured aneurysm of a cerebral artery, arteriovenous malformation, vasculitis, and cerebral artery dissection.
  • Hemorrhage in an arterial territory indicates hemorrhagic infarction. Multiple stages of bleed in the same hematoma with a fluid level are seen in anticoagulation-induced hemorrhage.
How well did you know this?
1
Not at all
2
3
4
5
Perfectly
20
Q

A 65-year-old male patient is being evaluated for a progressive headache of 1-month duration. Recently he is having multiple episodes of vomiting as well. CT head revealed a large ill- defined enhancing lesion in the frontal lobe with a hypodense center and perilesional edema. MR spectroscopy revealed the choline peak in the margins of the lesion. What is the most probable diagnosis in the patient?
1. Abscess
2. Hydatid cyst
3. Glioblastoma
4. Metastasis

A

3. Glioblastoma

  • The lesion which is ill-defined, enhancing after contrast administration and having center hypodensity (suggesting necrosis) is characteristic of GBM.
  • The presence of choline peak in MR spectroscopy (due to damage to the cell membrane) also suggests GBM.
  • Rapid onset of headache in an otherwise healthy patient and typical radiological findings are more suggestive of GBM in the patient.
  • The abscess has a lactate peak in MR spectroscopy with significant perilesional edema. Hydatid cyst has a characteristic laminated membrane, scolex and hydatid sand in radiological imaging.
How well did you know this?
1
Not at all
2
3
4
5
Perfectly
21
Q

A 45-year-old man presents with refractory migrainous headaches predominantly in his occipital region. A trial of botulinum toxin injection is performed with good pain relief. The decision to undertake the neurectomy of bilateral occipital nerves is made. He has had moderate relief for two months but is followed by the recurrence of his headaches, which are now high in intensity and are affecting his quality of life. Repeat treatment with botulinum toxin is not effective. What is the most appropriate next step in the management of this patient’s condition?
1. Opioid therapy
2. Repeat botulinum injection
3. Repeat neurectomy procedure
4. Occipital nerve stimulation

A

4. Occipital nerve stimulation

  • Neuromodulation is done when there is a failure of medical management or peripheral neurolysis is not feasible or has failed. It is done by placing electrodes around dorsal cervical nerves in the suboccipital region and connecting them first to a trial stimulator, and if successful, to a permanently implantable pulse generator.
  • Occipital nerve stimulation (ONS) can have benefits for pain in the distribution of both occipital and trigeminal nerves through the modulatory activity of the trigeminal-cervical complex, consisting of trigeminal nucleus and portions of upper three cervical dorsal nerves.
  • It can result in 30 to 50% of patients having more than 50% pain relief. Though this procedure has high rates of lead readjustments (around 50%), that is minimally invasive.
  • The repeated injection of botulinum toxin fails to provided benefits owing to the development of resistance. The chronic use of an opioid is also not advisable because of the high risks of dependency. The repeat neurectomy of the nerve is not justifiable unless there is evidence of a neuroma formation.
How well did you know this?
1
Not at all
2
3
4
5
Perfectly
22
Q

A 9-year-old male child presented for follow up with his pediatrician. He now comes in with complaints of severe headaches. He was seen a week ago for a productive cough with greenish sputum and fever. Blood works done pointed to a bacterial infection. He was then prescribed medications for his symptoms. On the current examination, he was awake, alert, and seems anxious. He would constantly rub his head. His pupils were isocoric, equally reactive to light, and extraocular muscles were equal with no complaints of diplopia. Funduscopic examination shows bilateral papilledema. The rest of the neurologic examination was normal. A plain head CT reveals normal results. Which of the following medications was the child most likely prescribed?
1. Trimethoprim
2. Erythromycin
3. Minocycline
4. Amoxicillin

A

3. Minocycline

  • Tetracyclines can cause pseudotumor cerebri. In infants, prior to fusion of the sutures, it can lead to bulging fontanels.
  • Tetracyclines should not be used in children 8 years, as they can lead to permanent teeth discoloration. If used long term, tetracyclines may produce microscopic brown-black discoloration of the thyroid. Rarely, abnormal thyroid function occurs.
  • Discontinuing the offending medication is recommended.
  • Treatment for pediatric pseudotumor cerebri is the same as in adults. Simply use weight-based doses for each medication.
How well did you know this?
1
Not at all
2
3
4
5
Perfectly
23
Q

A 37-year-old male crashes his motorcycle at a very high rate of speed into a bridge embankment. He was un- helmeted and intoxicated. Emergency medical services (EMS) arrived on the scene and found the patient hypotensive with a Glasgow coma scale of 8, consistent with a severe brain injury. The EMS crew performs endotracheal intubation, establishes intravenous crystalloid resuscitation, immobilizes the patient with a cervical collar on a long spine board, and transports the patient to the nearest trauma center. The patient now arrives in the trauma bay and is emergently evaluated. The patient remains hypotensive with a blood pressure of 80/60 mmHg and a heart rate of 120 beats per minute, consistent with class III hemorrhagic shock. On evaluation, he is found to have a blunt descending thoracic aortic dissection and left- sided traumatic subarachnoid hemorrhage with associated traumatic subdural hemorrhage with 10 millimeters of left to right shift with a dilated, non-reactive left pupil. Which of the following is the next best step in managing this patient?
1. Give mannitol and emergent right-sided decompressive hemicraniectomy
2. Resuscitate the patient to improve blood pressure to normal and emergent left-sided hemicraniectomy with the delayed repair of aortic injury
3. Resuscitate the patient to improve blood pressure to normal and emergent repair of the aortic injury
4. Aggressive medical management of the intracranial hypertension and aortic injury in the intensive care unit

A

2. Resuscitate the patient to improve blood pressure to normal and emergent left-sided hemicraniectomy with the delayed repair of aortic injury

  • Subdural hemorrhages greater than 10 millimeters of shift with evidence of a dilated non-reactive or blown pupil are often treated with decompressive operative therapy.
  • High-energy blunt thoracic injury often results in predictable injury patterns in which patients present with sternal, 1st or 2nd rib, and scapular fractures. When these are found, one should be highly suspicious of concomitant vascular injuries. When the thoracic aorta is injured, it predictably occurs within two centimeters distal to the take-off of the left subclavian artery where the ductus arteriosus is found, which is the interface of the fixed and mobile aorta.
  • Competing traumatic injuries such as major vascular and severe traumatic brain injuries are considered clinically challenging. Prevention of hypotension and hypoxia with alleviation of intracranial hypertension are the mainstays of this patient’s care. However, as intracranial pressure rises, cerebral perfusion pressure falls such that the only way to mitigate this, besides surgical decompression, is to raise the mean arterial pressure.
  • The concomitant traumatic aortic dissection is temporized until definitive care by medically keeping the blood pressure lower than normal with beta-blockade and/or calcium channel blockers. Here lies the conundrum in management where the traumatic brain injury needs high blood pressure and the aortic dissection needs low blood pressure. Traditionally the traumatic brain injury takes precedence and is surgically managed, while the traumatic aortic dissection is often managed in a delayed fashion.
How well did you know this?
1
Not at all
2
3
4
5
Perfectly
24
Q

A 45-year-old woman presents to the clinic with new-onset urinary incontinence, bilateral lower extremity weakness, and severe back pain with movement. She has a known history of metastatic stage IV breast cancer. MRI reveals a lytic metastatic lesion in the T2 spine, >50% vertebral collapse, and compression on the spinal cord. Physical examination reveals 3+ patellar reflexes bilaterally. The patient is started on dexamethasone. What is the most appropriate treatment recommendation?
1. Supportive care only
2. Palliative radiotherapy only
3. Kyphoplasty and adjuvant radiotherapy
4. Surgical stabilization and adjuvant radiotherapy

A

4. Surgical stabilization and adjuvant radiotherapy

  • The SINS scoring system is used by spine surgeons to assess the need for surgical stabilization of metastasis to the spine, even when palliative treatment is indicated. The greater the total score across six categories, the more likely the patient requires surgical stabilization. A SINS score >7 warrants at least consideration of stabilization for metastatic spine lesions that likely cause instability.
  • In this case, the patient’s SINS score is at least 12, even without having information about all categories, and indicates impending spinal instability.
  • Metastatic lesions to the vertebrae can lead to spinal cord compression, resulting in loss of bowel and bladder function, weakness, numbness or sensory deficits, and hyperreflexia.
  • Kyphoplasty would be inappropriate in the setting of a tumor involving the spinal canal with evidence of cord compression. Supportive care would be an option for a patient with poor functional status before presentation or who refuses all intervention.
How well did you know this?
1
Not at all
2
3
4
5
Perfectly
25
Q

A 6-year-old child is brought with a history of sudden onset loss of consciousness following a headache that had been present for the past 5 days. On examination, the Glasgow Coma Scale score is E2V2M5. He undergoes an emergency CT scan of the brain, which shows obstructive hydrocephalus with a mass in the pineal region. He undergoes an emergency ventriculostomy of the third ventricle. The cerebrospinal fluid turns to be positive for placental alkaline phosphatase. What is the probable diagnosis?
1. Germinoma
2. Choriocarcinoma
3. Endodermal sinus tumor
4. Teratoma

A

1. Germinoma

  • The patient has a pineal region tumor with obstructive hydrocephalus, which is causing the drop in the Glasgow Coma Scale score.
  • Since the CSF is positive for placental alkaline phosphatase (PLAP), the most probable diagnosis is a germinoma.
  • PLAP can also be positive in the serum of the patient.
  • Beta-human chorionic gonadotropin is positive in choriocarcinoma.
How well did you know this?
1
Not at all
2
3
4
5
Perfectly
26
Q

A 17-year-old male high school football player presents to the emergency department after sustaining a significant collision during a game. While running at full speed, he caught a pass, and as he turned to continue running up the field collided with an opposing player. He was driven backward and landed on his back; however, he did not get up immediately. As the on-field medical team reached him, he seemed to be awake. His Glasgow coma scale was 14. He was appropriately immobilized and transported to the emergency department, where the CT scan of the head was done and was reported as normal. Which findings do not warrant the athlete’s removal from upcoming football games?
1. Loss of consciousness
2. Dizziness, headache, or nausea lasting longer than 15 minutes
3. His Glasgow coma scale of 14
4. Prior baseline neuropsychological testing demonstrating a low score

A

4. Prior baseline neuropsychological testing demonstrating a low score

  • A patient who has suffered a brain injury may complain of a headache, nausea, memory loss, dizziness, blurry vision, confusion, fatigue, photophobia or phonophobia, motor or sensory loss, poor hand-eye coordination, irritability, or depression. Patients with these symptoms should be prohibited from participating in activities and immediately evaluated.
  • The Glasgow coma scale is a validated tool for rapidly assessing a person’s level of consciousness. It has three components: eye-opening, verbal response, and motor response. Points are assigned based on the patient’s response. The best score is 15, and anything less than eight is considered severe head injury/coma. This athlete scored in the mild category with a score of 14 to 15. The tool is not validated to predict the risk for a second impact syndrome and should not be used to allow an athlete to participate once a concussion has been suspected or diagnosed.
  • Neuropsychologic testing should be performed both initially and during follow-up evaluation of a patient with a history of concussion. However, the early baseline reading that is often performed on athletes to have a standard for comparison with a low score does not warrant prohibition from participation in activities and sports. Some athletes may try and get an initial low score, so when they repeat the test and get a low score, it will be close to their baseline.
  • Due to the increased fragility of the brain following an initial head injury, only a minimal amount of force is required on a subsequent incident to result in irreparable damage. Therefore, a high index of suspicion for second injury syndrome is warranted for all patients with a known history of head injury. Athletes with a head injury should not be allowed to return to participation until completing an approved return to play protocol.
How well did you know this?
1
Not at all
2
3
4
5
Perfectly
27
Q

A 4-year-old girl presents with a wide face, loss of spinal curvature, and acanthosis nigricans around the lips and in the axillae. These findings suggest the presence of which genetic mutation?
1. Chromosome 22q
2. Chromosome 21
3. FGFR3
4. Trisomy of chromosome 3

A

3. FGFR3

  • Crouzon syndrome is characterized by facial deformations at or near birth.
  • The physical findings include a wide face secondary to the obliteration of the coronal and sagittal sutures.
  • Findings include hypertelorism, wide and high forehead, malocclusion, short stature, and the absence of physiologic spinal curvature.
  • Acanthosis nigricans of the axillary fossa is seen in those with FGFR3 mutations.
How well did you know this?
1
Not at all
2
3
4
5
Perfectly
28
Q

A 65-year-old hypertensive patient presented to the emergency department with altered sensorium and left-sided hemiparesis. CT scan showed a right-sided basal ganglionic bleed. The patient was managed conservatively along with antihypertensive drugs for the optimization of blood pressure. The patient suddenly deteriorated and had the blood pressure of 80/60 mm Hg. His pupils were both equal and reactive to light. Repeat CT images showed no progression of the hematoma but there was evidence of multiple infarcts in the watershed zones within the brain. What is the mechanism for the neurological deterioration in the patient?
1. Intracranial hypertension
2. Seizure
3. Altered cerebral autoregulation
4. Rebleed

A

3. Altered cerebral autoregulation

  • In chronic hypertensive patients, there is a rightward shift of the cerebral blood flow autoregulatory curve.
  • The lower limit of the autoregulation curve is increased thereby a sudden drop in blood pressure can lead to diminished cerebral blood flow thereby resulting in ischemia.
  • It is therefore recommended to maintain the systolic blood pressure of these patients at around 130mm Hg. This minimizes the risk of rebleed as well as protects from the effects of hypotension.
  • Intracranial pressure can not increase without an increase in the size of the hematoma in the CT images. Patients with seizures will have visible abnormal body movements and the uprolling of eyes and frothing from the mouth, which was absent in this patient.
How well did you know this?
1
Not at all
2
3
4
5
Perfectly
29
Q

A CT-scan of a 65-year-old woman shows an acute ischaemic infarction of the white matter structure situated in the inferomedial part of each cerebral hemisphere of the brain. The blood supply to the inferior part of the posterior limb of this area of the brain is supplied by which of the following?
1. Posterior cerebral artery
2. The anterior choroidal artery
3. The recurrent artery of Heubner
4. The lateral lenticulostriate arteries

A

2. The anterior choroidal artery

  • The white matter structure situated in the inferomedial part of each cerebral hemisphere of the brain is the internal capsule.
  • The inferior part of the posterior limb of the internal capsule is supplied by the perforating branches anterior choroidal artery.
  • The superior levels of the anterior limb, genu, and posterior limb get their supply from perforating arteries of the middle cerebral artery.
  • The inferior levels of the anterior limb obtain their blood supply by the Heubner artery and perforating arteries of the anterior cerebral artery and the inferior levels of the genu get supplied by perforating arteries of the internal carotid artery and proximal perforating arteries of the anterior choroidal artery.
How well did you know this?
1
Not at all
2
3
4
5
Perfectly
30
Q

A 26-year-old male patient is receiving treatment for acute lymphoblastic leukemia with central nervous system involvement and is recommended a ventricular access catheter for intrathecal chemotherapy. The patient is taken to the operating room, and an Ommaya reservoir is inserted. The intraventricular catheter was cut at 6.5 cm as per the operative report. During the third session of chemotherapy, the oncologist attempts to aspirate to obtain cerebrospinal fluid (CSF) before injecting the medication, but CSF could not be obtained. There is no redness around the catheter, and the patient shows no signs of meningeal irritation. What is the most likely cause of this finding?
1. Ventricular catheter infection
2. Choroid plexus obstruction
3. Liliequist membrane adherence
4. Septum pellucidum obstruction

A

2. Choroid plexus obstruction

  • The length of the catheter is cut as estimated according to preoperative imaging (roughly 5-5.5 cm when measured at the inner table of the calvarium) and attached to the base of the reservoir with a silk tie. This length allows the tip of the catheter to be positioned on the floor of the anterior horn of the lateral ventricle.
  • A longer intraventricular catheter, as in this patient, is often associated with obstruction due to the choroid plexus at the foramen of Monroe.
  • The correct placement of the catheter can be verified with an image guidance system.
  • The distal end of the intraventricular catheter does not usually get obstructed by septum pellucidum as it can not occlude its orifices or by the membrane of Liliequist, which is outside the ventricles. An infection will not occlude a catheter unless the patient has significant debris from the ventriculitis. Mechanical failure is very uncommon with modern Ommaya reservoirs.
How well did you know this?
1
Not at all
2
3
4
5
Perfectly
31
Q

A six year old female complains of low back pain which is worse after physical activity and improved when relaxing. She does not have any changes in bowel or bladder function or strength. A plain film X-ray is reported as “Grade II spondylolisthesis.” What is the most likely type of spondylolisthesis?
1. Dysplastic
2. Isthmic
3. Degenerative
4. Traumatic

A

1. Dysplastic

  • Dysplastic spondylolisthesis is of a congenital origin and affects children.
  • Isthmic spondylolisthesis is caused by a defect in the pars interarticularis and is most common in those in their second decade and older.
  • Degenerative spondylolisthesis typically affects older individuals.
  • Traumatic spondylolisthesis occurs after high impact injuries such as hyperflexion with compression or axial rotation in a motor vehicle collision.
How well did you know this?
1
Not at all
2
3
4
5
Perfectly
32
Q

A 65-year-old male patient was diagnosed with glioblastoma multiforme and underwent surgical resection followed by chemoradiotherapy. Repeat MRI done six weeks after initiation of therapy showed the increase in size with reduced cerebral blood volume, low choline ratio Cho/NAA ratio of less than1.48, increased lactate peak, increased lipid peak, and elevated ADC values due to cell death. On evaluation, the patient denies any new symptoms. What is the next best step in management?
1. Discontinue the intensive treatment due to the progression of the disease and recommend palliative measures.
2. Continue with therapy followed by repeat imaging in 6-8 weeks.
3. Proceed with surgery again.
4. Get a PET scan to assess for progression of the disease.

A

2. Continue with therapy followed by repeat imaging in 6-8 weeks.

  • Continuing with therapy followed by repeat imaging in 2 months is recommended as the imaging finding is likely a treatment- related effect called pseudoprogression. Pseuoprogression is usually asymptomatic, which is the case in this scenario.
  • Pseudoprogression is a phenomenon that is usually seen four to six weeks after the completion of chemoradiotherapy.
  • Pseudoprogression is a treatment-related effect with imaging findings similar to tumor progression. It usually occurs within three months of completion of chemoradiation in glioblastoma patients.
  • Distinguishing between pseudoprogression due to treatment and true progression of the disease is important to avoid inappropriate discontinuation of treatment in patients.
How well did you know this?
1
Not at all
2
3
4
5
Perfectly
33
Q

A 46-year-old male was brought to the emergency department with injury to his cervical spine following a motor vehicle collision. On physical exam, his neurological status was normal. Computerized tomography (CT) scan of the head and cervical spine revealed a C1 burst fracture with C1 and C2 subluxation. The subluxation was reducible. Given the underlying instability, a C1 and C2 fusion were to be performed in the prone position with transarticular Magerl screw fixation. Which of the following is a relatively common vascular complication associated with this procedure?
1. Internal carotid artery injury
2. Vertebral artery Injury
3. Posterior inferior cerebellar artery injury
4. Anterior spinal artery injury

A

2. Vertebral artery Injury

  • C2 transarticular screw represents a multiple level instrumentation option. The screw is applied at a trajectory 0 to 15 degrees medial from the starting point at the inferomedial angle of C2-C3 facet.
  • However, this procedure provides the highest rate of possible vertebral artery injury.
  • Detailed CT scan evaluation of the anatomy of C1 needs to be obtained before any surgical intervention to identify any abnormal course or anomaly of the vertebral artery (e.g., high riding vertebral artery) before such arthrodesis.
  • Vertebral artery injury on one side should preclude the surgeon from the insertion of a similar screw contralaterally, as bilateral vertebral artery injury can lead to devastating brain stem stroke
How well did you know this?
1
Not at all
2
3
4
5
Perfectly
34
Q

A 33-year-old male presents with complaints of neck pain and occipital headaches. He reports a history of whiplash injury 3 years ago. He says moving his neck makes his headaches worse but denies any history of nausea/vomiting or photophobia/phonophobia. Magnetic resonance imaging (MRI) brain without contrast and neurological examination of the patient was normal. What is the next preferred management?
1. Magnetic resonance imaging of the cervical spine
2. Computed tomography scan of the brain
3. Cerebrospinal fluid analysis
4. Naproxen

A

1. Magnetic resonance imaging of the cervical spine

  • The patient describes a clinical case of occipital headaches with neck pain with a history of whiplash injury to the neck. This is typical of cervicogenic headaches.
  • Even though the diagnosis of cervicogenic headaches is clinical, magnetic resonance imaging (MRI) cervical spine should be ordered to identify the source of pathology in the C-spine such as disc herniation, degenerative disc disease, etc.
  • MRI cervical spine is necessary to identify the accurate level of pathology in the c-spine for possible interventional therapy, including interventional procedures/surgery.
  • The patient’s symptoms are not consistent with a migraine, so naproxen is unnecessary. MRI brain with contrast would offer no additional diagnostic advantages for the patient in a setting of cervicogenic headaches. Lumbar puncture is not indicated for the diagnosis of cervicogenic headaches.
How well did you know this?
1
Not at all
2
3
4
5
Perfectly
35
Q

A 29-year-old male with HIV for the past 8 years complains of a 2-month history of visual changes. He has been having transient flashes together with a distortion of a portion of his left visual field. Routine electroencephalography (EEG) identifies a seizure focus within the right occipital lobe. Magnetic resonance imaging (MRI) shows a 3 cm right occipital lobe lesion that shows rim enhancement post-contrast. Cerebrospinal fluid polymerase chain reaction analysis is positive for Epstein-Barr virus. The patient is scheduled for a stereotactic biopsy. Which of the following is the most likely diagnosis?
1. Astrocytoma
2. Primary central nervous system lymphoma
3. Progressive multifocal leukoencephalopathy
4. Kaposi sarcoma

A

2. Primary central nervous system lymphoma

  • Central nervous system lymphoma is seen in up to 6% of patients with AIDs and is the second most common mass lesion of the central nervous system in the AIDs population. Immunocompromised individuals can present with single or multiple lesions.
  • Presentation is often related to the location of the lesion. Occipital lobe lesions usually result in visual field defects. Seizures are more commonly seen in AIDs related central nervous system lymphomas.
  • The Epstein-Barr virus is associated with 100% of AIDs related central nervous system lymphomas. Magnetic resonance imaging is the best initial test used in diagnosis. In immunosuppressed individuals, enhancement is often irregular or rim-like due to the presence of central necrosis. Stereotactic brain biopsy is the gold standard for diagnosis.
  • Diffuse large B cell lymphoma is the most common histological variant seen. High-dose methotrexate has demonstrated efficacy in the treatment of AIDs related central nervous system lymphoma.
How well did you know this?
1
Not at all
2
3
4
5
Perfectly
36
Q

A 17-year-old male patient presents to the emergency department with progressively worsening headaches, nausea, and vomiting for the past week. The patient is alert and oriented. However, he has difficulty looking up, the pupils are unreactive to light, and he has nystagmus with an upward gaze. He cannot close eyelids completely, and his eyes have a “sun-setting” appearance. Brain MRI was performed and showed a pineal region mass with obstructive hydrocephalus. What anatomical structure is most likely affected in this patient, causing the difficulty looking up?
1. Periaqueductal gray area
2. Rostral interstitial nucleus of medial longitudinal fasciculus and interstitial nucleus of Cajal
3. Pretectal nucleus
4. Medial lemniscus

A

2. Rostral interstitial nucleus of medial longitudinal fasciculus and interstitial nucleus of Cajal

  • Tumors of the pineal region cause Parinaud syndrome due to compression of the upper brainstem (midbrain).
  • This syndrome includes; paralysis of upwards gaze, pupils with light-near dissociation, convergence-retraction nystagmus, eyelid retraction, and conjugate down-gaze.
  • Paralysis of the upwards gaze and conjugate down-ward gaze is caused by compression of the vertical gaze center at the rostral interstitial nucleus of medial longitudinal fasciculus.
  • Light-near dissociation is caused by the compression of the pretectal nucleus. Convergence-retraction nystagmus and eye- lid retraction (collier’s sign) is caused by damage to the supranuclear fibers of the third nerve at the posterior midbrain.
How well did you know this?
1
Not at all
2
3
4
5
Perfectly
37
Q

A 76-year-old female with osteoporosis presents with constant, sharp pain localized to her lower back for the past week. She has a history of a minor slip and fall two weeks ago. She does not have any neurological deficits. A lumbar spine radiograph demonstrates an age-indeterminate, wedge-shaped vertebral compression fracture at the L1 level, which is confirmed with a CT scan. A magnetic resonance imaging of the spine shows no evidence of an associated lesion. She is being planned for kyphoplasty. Which of the following is a potential complication of this treatment modality?
1. Ischemic stroke
2. Myocardial infarction
3. Rhabdomyolysis
4. Pulmonary embolus

A

4. Pulmonary embolus

  • A documented potential complication of vertebroplasty and kyphoplasty is pulmonary embolus.
  • Overzealous injection of polymethyl methacrylate (PMMA) cement can lead to particles entering the vertebral venous plexus leading to eventual embolization to the lungs.
  • Vertebroplasty and kyphoplasty are done under direct, fluoroscopic visualization to avoid extra-vertebral cement administration.
  • Other potential complications include infection, spinal hematoma, and nerve damage. Antibiotic prophylaxis, coagulation panels, and fluoroscopy help to minimize the risk of complications.
How well did you know this?
1
Not at all
2
3
4
5
Perfectly
38
Q

A 45-year-old woman had an asymptomatic middle cerebral artery aneurysm clipped successfully after she had an MRI scan, obtained for chronic headaches, which showed the incidental aneurysm. Her younger sister had a ruptured vertebrobasilar aneurysm, which presented with subarachnoid hemorrhage (SAH). Their mother collapsed and died suddenly at age 47 before she could get to a hospital. What evaluation could be recommended for this woman?
1. Renal ultrasound
2. Chest CT scan
3. Holter monitor
4. Transthoracic echocardiogram

A

1. Renal ultrasound

  • When multiple members of the family are affected with cerebral aneurysms, it should raise the suspicion of autosomal polycystic kidney disease.
  • The incidence of aneurysms in these cohorts of patients is approximately 11%. Though the incidence is comparable to the general population, these patients have a high risk of developing progressive renal cysts as well.
  • The prime defect is the mutation in the PKD1 and PKD2 genes. This leads to the formation of multiple cysts within various organs such as the liver and kidney. It also predisposes to the formation of multiple intracranial aneurysms.
  • Other syndromes associated with multiple intracranial aneurysms include Marfan syndrome, hypereosinophilic syndrome and Diamond-Blackfan syndrome.
How well did you know this?
1
Not at all
2
3
4
5
Perfectly
39
Q

A 43-year-old man falls from a ladder hitting his head on a concrete floor. He is unconscious in the emergency department. His eyes are closed to painful stimuli; he is making no verbal effort and is flexing on the right of his body to pain, with no movement of the left side. His left pupil is dilated with minimal reaction to light. The right pupil is normal. He has labored breathing with stertorous sounds. His blood pressure is 180/95 mmHg, pulse 55/min, and respiratory rate 8/min. He is immobilized in a cervical collar, with blocks and is on a trauma mattress. Which of the following is the next best step in the management of this patient?
1. IV mannitol
2. Bedside placement of a left-sided external ventricular drain
3. Intubate and ventilate the patient with a target paCO2 of 4.5 to 5.0 kPa
4. Placement of arterial and central lines to target a MAP of 60 to 70 mmHg

A

3. Intubate and ventilate the patient with a target paCO2 of 4.5 to 5.0 kPa

  • The priority next step in the management of the patient is to secure the airway with intubation. There is level III evidence that patients with TBI and a GCS of 8 or less should be intubated.
  • Avoidance of hypoxia and control of paCO2 is vital. There is level III evidence that O2 saturations 90% in TBI significantly raise mortality. The target paCO2 of 4.5 to 5 kPa is ideal.
  • An ABC approach is vital in trauma patients (airway, breathing, circulation). After securing the airway and ventilating the patient, a target MAP of 60 to 70 mmHg is recommended with level III evidence to reduce mortality in TBI.
  • The administration of osmotic therapy in TBI is indicated in an acutely deteriorating patient with signs of herniation. It would be appropriate to administer mannitol to this patient while obtaining a CT scan, but securing the airway and ventilating is the first priority. Blind placement of an EVD without CT imaging in this patient would not be appropriate.
How well did you know this?
1
Not at all
2
3
4
5
Perfectly
40
Q

A 45-year-old man presented to the hospital with sudden pain in his right eye. The pain started abruptly while he was working in his office. He describes the pain as retroorbital, and he is not able to elevate his right eyelid. On examination, he is photophobic with a blood pressure of 170/95 mmHg. There is complete ptosis of the affected eye, and manual elevation of the lid reveals a downward and outward deviation of the eye. In addition, the right-sided pupil is large and partially reactive to light. His past medical record shows that he has been frequently seen for oral and genital ulcers. His ophthalmologist diagnosed him with posterior uveitis six months ago. Laboratory investigations suggest an erythrocyte sedimentation rate of 95 mm/hr. What investigation would help the most in establishing the diagnosis?
1. Magnetic resonance imaging with IV contrast
2. Magnetic resonance angiography of cerebral vessels
3. Fundus fluorescein angiography
4. Duplex ultrasound of the right side of the neck

A

2. Magnetic resonance angiography of cerebral vessels

  • Behcet disease is an auto-inflammatory systemic vasculitis of unknown etiology and is characterized by mucocutaneous manifestations including recurrent oral and genital ulcerations, ocular manifestations especially chronic relapsing uveitis and systemic vasculitis involving arteries and veins of all sizes.
  • This patient appears to have features suggestive of posterior communicating artery aneurysm. The involvement of vasa vasorum (vasculitis) may result in the formation of aneurysms in the arteries.
  • Venular involvement and formation of pulmonary and arterial aneurysms are unique to Behcet disease.
  • The most helpful test in the diagnosis of cerebral aneurysms is contrast angiography. Management of aneurysms that have already bled is endovascular coiling or surgical clipping.
How well did you know this?
1
Not at all
2
3
4
5
Perfectly
41
Q

A 50-year-old man with a history of trauma presents with delayed CSF rhinorrhoea. He has a history of diabetes mellitus and hypertension. On a thin-cut non-contrast CT head, he is diagnosed with a temporal fracture and is taken to the operating room for surgical repair of the dural defect. The operating neurosurgeon decides to put a lumbar drain in the postoperative period. On day 3 following surgery, his GCS dips to E3V2M5 with the left side pupil nonreactive and dilated. Non-contrast CT head shows a left fronto-temporo-parietal hyperdense subdural collection with a midline shift of 11 mm. What is the likely pathophysiology behind this event?
1. Injury to draining vein during the temporal dura repair
2. Injury to bridging veins due to CSF over-drainage
3. Delayed presentation of head trauma
4. Patient has a history of hypertension

A

2. Injury to bridging veins due to CSF over-drainage

  • CSF over-drainage is a common complication following CSF diversion procedures like lumbar drain insertion.
  • CSF over-drainage leads to sagging of the brain parenchyma which can lead to injury to bridging veins and result in subdural hematoma.
  • If a patient presents with acute deterioration of GCS and evidence of herniation (pupillary dilation), urgent evacuation of subdural hematoma should be performed.
  • Controlled CSF drainage and preventing intracranial hypotension is very important following any CSF diversion procedure.
42
Q

A 65-year-old female with a history of osteoporosis and kyphoscoliosis presents to the clinic for follow up. The patient reports worsening back pain, refractory to oral medications. She has tried a course of physical therapy in the past without lasting benefit. Her home health aid says that the patient has had a progressive decline in mobility over the past several months. The patient had previously ambulated with a rolling walker but recently had become more reliant on her wheelchair. Physical examination reveals a diminished bilateral lower extremity strength and sensation to light touch. The patient admits that she did not seek medical attention earlier, as she attributed her decline to seasonal changes and her age. In addition to obtaining new imaging of the spine, which of the following is the next best step in management?
1. Spinal cord decompression
2. Evaluate for a custom wheelchair to better suit her new functional status
3. Restart physical therapy to address lower extremity weakness
4. Epidural steroid injection

A

1. Spinal cord decompression

  • Given the patient’s history of kyphoscoliosis and osteoporosis, her complaints of decreased mobility, lower extremity weakness, and loss of sensation are likely due to spinal cord compromise secondary to progression of spinal defects. Compression of the spinal cord can lead to devastating complications such as paraparesis or paraplegia and warrants referral for surgical evaluation.
  • Operative management may be indicated in those patients who have failed conservative therapy, experience intractable pain, or have an onset of neurologic changes.
  • Neurologic manifestations can include changes in strength, sensation, reflexes, or bowel/bladder control.
  • Given the patient’s history of osteoporosis and worsening back pain with neurological signs, an epidural steroid injection is contraindicated.
43
Q

A 45-year-old male was brought to the ED for generalized seizures. He has been complaining for the past six months of dry cough, fatigue, and shortness of breath. His wife claims that for the past three months, he has become increasingly irritable and forgetful. He would often misplace his wallet and house keys. A month ago, he presented with Bell’s palsy. On examination, he was drowsy but arousable. He was oriented to place and person but not to time. He was able to follow commands, there is peripheral facial palsy on the right, and he moves all extremities equally to command. Babinski was positive bilaterally. An MRI shows hydrocephalus with diffuse meningeal enhancement. Blood tests show an elevated erythrocyte sedimentation rate (ESR) and angiotensin-converting enzyme (ACE) along with increased liver function tests. What is the best next step in management?
1. Corticosteroids
2. Neurosurgery consult
3. No treatment is needed because the condition will resolve on its own
4. Kveim test

A

1. Corticosteroids

  • Corticosteroids are the mainstay of treatment for neurosarcoidosis. The majority of patients with sarcoidosis do not have symptoms. Neurosarcoidosis may be life-threatening and tends to occur in cases with systemic involvement.
  • Symptoms of neurosarcoidosis may be multifocal, commonly seen first in the cranial nerves. The facial nerve is the most common cranial nerve affected, which may mimic Lyme disease. Psychiatric and cognitive symptoms may occur in patients with diffuse meningeal inflammation associated with neurosarcoidosis.
  • MRI findings of meningeal enhancement may translate to seizures in a clinical setting. An F-18 fluorodeoxyglucose positron emission tomography (PET) scan can be helpful in identifying areas with active granulomatous inflammation.
  • In cases of a meningeal, parenchymal mass lesion or myopathy or neuropathy, prednisone should be given for at least 4 weeks. For deteriorating patients, intravenous (IV) methylprednisolone is given for 3 days followed by prednisone for 2 to 4 weeks.
44
Q

A 65-year-old female patient presents to the emergency department with sudden onset right-sided weakness. A quick examination shows a preferential gaze to the left, facial droop on the right, and dense hemiplegia on the right. She underwent a thrombectomy. There is an improvement in her symptoms after the procedure. Examination reveals primary gaze at midline, a shallow right nasolabial fold, and a drift on the right upper and lower extremity. A 24-hour post-procedure magnetic resonance imaging (MRI) shows a small diffusion-weighted image (DWI) signaling lesion with a corresponding signal drop in the apparent diffusion coefficient (ADC) map on the left parietooccipital region. Which of the following tests would be difficult for this patient?
1. Facial recognition of familiar people
2. Naming items placed on the hand
3. Naming common items that are visually introduced
4. Pick named items from a selection

A

3. Naming common items that are visually introduced

  • Agnosia is the inability to recognize familiar items visually, so presenting the items and asking the patient to name them would be the best screen.
  • The deficit cannot be explained by memory, attention, language problems, or unfamiliarity with the stimuli.
  • Visual agnosia refers to an impairment in recognizing visually presented objects, despite otherwise normal visual field, acuity, color vision, brightness discrimination, language, and memory.
  • Apperceptive visual agnosia is an abnormality in visual perception and discriminative process, despite the absence of elementary visual deficits. Apperceptive visual agnosia is typically associated with lesions to the parietal and occipital cortex.
45
Q

A 33-year-old woman presents to the clinic complaining of pain left-sided shoulder and neck region. She states the pain is localized to that region and has been ongoing for about two weeks. She denies any recent falls or trauma. On examination, force is adequate in both upper extremities, but the left biceps reflex is diminished compared to the right side. Magnetic Resonance Imaging shows a C4-C5 left herniated disc with foraminal stenosis. Motor and sensory nerve conduction studies are performed, including electromyography. Fibrillation potentials are seen in the C5 cervical paraspinal muscles of the left side. Which of the following motor nerve conduction findings is most consistent with the patient’s underlying diagnosis and clinical scenario?
1. Prolonged latency
2. No change in latency
3. Decreased amplitude
4. Increased amplitude

A

2. No change in latency

  • Latency is a measure of the speed of the fastest muscle fibers. There is no change in latency in cervical radiculopathy.
  • As with sensory nerve conduction studies, the sensory and motor nerve conduction study will be normal in cervical radiculopathy. Latency and amplitude are usually normal.
  • These normal findings occur because only a fraction of nerve fascicles within the nerve root are affected. However, axon loss ensues if there is sufficient root compression where distal parts of axons are not continuous with the cell body.
  • Amplitude is a measure of the magnitude of muscle force. It is unchanged in typical cervical radiculopathy. However, spontaneous activity of the paraspinal muscles is often seen on EMG within the first five to seven days.
46
Q

A 26-year-old patient with Down syndrome works full-time washing dishes. She has started falling when she walks long distances and has developed daytime encopresis. Her recent laboratory tests were WBC: 8700 cells per mm^3, hemoglobin: 12 mg/dL, platelet: 176,000 cells per microliter, sodium: 138 mEq/L, potassium: 4.8 mEq/L, fasting blood sugar: 98 mg/dL, BUN: 20 mg/dL, creatinine: 1.1 mg/dL. Her past surgical history is unremarkable. What is the next preferred management?
1. Conservative treatment with laxative therapy
2. Obtaining cervical CT and measuring the lateral overhang of the lateral masses of C1 on C2
3. Biofeedback schedule
4. Bone marrow aspiration

A

2. Obtaining cervical CT and measuring the lateral overhang of the lateral masses of C1 on C2

  • Changes in bladder and bowel function can be caused by cervical spine instability.
  • 10% to 30% of patients with Down syndrome have atlantoaxial instability with or without subluxation, but estimates vary.
  • Early symptoms may be challenging to get from the patients and may need to be inferred by refusing to participate in usual activities. Any change in motor skills or progressive paralysis should cause consideration of this diagnosis.
  • Although overflow incontinency might occur in those patients with mental retardation, which might respond to bowel management schedules, the newly onset encopresis without any previous history in the mentioned patient is unlikely.
47
Q

A 17-year-old female falls from 15 feet to a concrete floor. At the scene she is moaning, eyes closed, and localizing to pain. Pupils are equally round and reactive to light at 4 mm at the scene. She is transported to the hospital and deteriorates during transport. At arrival at the emergency department, she does not open eyes to noxious stimuli, is intubated and extensor postures in the upper extremities to noxious stimuli. Her right pupil is 8 mm and non-reactive, and her left pupil is 3 mm and reactive. A computed tomography scan of her head shows significant right cerebral edema and uncal herniation. A decision is made to give her mannitol while getting her to the operating room. Which of the following is the correct dose of mannitol to give?
1. 1 mg/kg over 30 minutes intravenously
2. 1 mg/kg over 30 minutes subcutaneously
3. 1 g/kg over 30 minutes intravenously
4. 1 g/kg over 30 minutes subcutaneously

A

3. 1 g/kg over 30 minutes intravenously

  • The dosing of mannitol to decrease intracranial pressure is 0.25 to 2 g/kg intravenous.
  • Mannitol should never be given subcutaneous or intramuscular.
  • Mannitol is typically given over 30 to 60 minutes.
  • It should be ensured patient has a response to mannitol with increased urine output.
48
Q

A 33-year-old man presents to the emergency department (ED) with C4-5 AO type B2 injury with ASIA B neurology. He sustained the injury following a motor vehicle accident 6 hours back. He is intubated, and his cervical spine is immobilized with a hard cervical collar. He has been planned for C4-5 anterior cervical discectomy and fusion. He does not have any other systemic injury. Which of the following is the next best step in the management of this patient?
1. High dose methylprednisolone sodium succinate - bolus dose followed by maintenance infusion for 23 hours.
2. Expectant management only.
3. High dose methylprednisolone sodium succinate - bolus dose followed by maintenance infusion for 48 hours.
4. A single bolus dose of high dose methylprednisolone sodium succinate.

A

2. Expectant management only.

  • The pathophysiology of spinal injury involves two stages - a. primary injury, which happens directly at the time of trauma; and b. secondary injury happening due to necro-inflammatory responses leading to cell death, tissue degeneration, the formation of a glial scar, axonal degeneration, and demyelination.a
  • Various pharmacological agents have been tried in the past to reduce the inflammatory responses causing secondary spinal injury. Over the past 30 years ago, methylprednisolone was believed to decrease lipid peroxidation-triggered secondary damage following spinal cord injury.
  • National Acute Spinal Cord Injury Studies II (NASCIS II) study recommended 30 mg/kg bolus dose of methylprednisolone sodium succinate on admission, followed by infusion at the rate of 5.4 mg/kg/h for the next 23 h in patients presenting less than 3 hours and infusion at the rate of 5.4 mg/kg/h for the next 48 h in patients presenting less than 8 hours.
  • However, no other randomized controlled studies could validate these findings of the NASCIS II trial. It is no longer recommended in the American Association of Neurological Surgeons (AANS) guidelines for the management of acute spinal cord injuries.
49
Q

A 41-year-old female gave birth to a dead fetus. An autopsy revealed a fetus with cyclopia, polydactyly, and a cleft palate. Which of the following additional abnormalities are likely to be present in this fetus?
1. Spasticity
2. Anencephaly
3. Myelomenignocele
4. Holoprosencephaly

A

4. Holoprosencephaly

  • This fetus likely has trisomy 13, or Patau syndrome. Patau syndrome is a chromosomal abnormality in which affected fetuses have an extra copy of chromosome 13.
  • Most patients with Patau syndrome will die in utero, so the diagnosis is usually made during autopsy. Of the affected infants that are born alive, the median survival is seven days.
  • Patau syndrome presents with severe intellectual disability, cleft lip/palate, holoprosencephaly, polydactyly, cutis aplasia, and congenital heart disease.
  • Holoprosencephaly is a disorder in which the forebrain of the fetus fails to separate into a left and right hemisphere. In mild to moderate cases, a cleft lip or palate may occur. In severe cases, affected patients can be born with cyclopia. Cyclopia is when both eyes are fused into a single midline structure.
50
Q

A 65-year-old female is brought to the emergency department after losing consciousness at home. Her husband states that she lost consciousness while reading the newspaper, became very rigid, and passed urine when unconscious. The patient regained consciousness after several minutes but remained confused and lethargic for two hours. The patient denies any prior episodes like this in the past. Her past medical history is relevant for hypertension, hyperlipidemia, and lumpectomy for locally advanced breast cancer over 1 year ago. Medications include aspirin, atorvastatin, and metoprolol. Vital signs are within normal limits. The patient is alert and oriented. Neurological examination demonstrates 4/5 right upper extremity weakness but is otherwise neurologically intact. What is the most appropriate initial management of the patient?
1. Computed tomography (CT) scan head without intravenous contrast
2. Computed tomography (CT) scan head with intravenous contrast
3. Magnetic resonance imaging (MRI) brain without intravenous contrast
4. Electroencephalogram (EEG)

A

1. Computed tomography (CT) scan head without intravenous contrast

  • Computed tomography (CT) of the brain is the most commonly employed imaging test in the emergent setting for patients with new-onset seizures. It is the preferred test in the emergency setting, given the speed at which it can be obtained. CT is used to rule out life-threatening pathologies such as intracranial hemorrhage or stroke but plays a limited role in the evaluation of seizures. It is reasonable to pursue a CT brain initially in this patient, given her right upper extremity weakness, which could be due to Todd paralysis or stroke. A non-contrast CT should be obtained as a first screen for either of these pathologies while a patient is in the emergency room.
  • Contrast-enhanced CT can demonstrate regions of abnormal enhancement within the brain but is less sensitive than MRI, and seldomly performed. CT angiography may be used to rule out vascular lesions such as aneurysm, dissection, and vasculitis, but these abnormalities rarely present with seizures.
  • Following CT imaging of the brain, if no acute abnormalities are discovered, an MRI should be obtained on a non-emergent basis. A late-life first-onset seizure is usually a sign of malignancy or stroke, as epilepsy is a disease that presents in childhood and early adulthood.
  • A brain MRI can be used to evaluate nearly any anatomic cause of seizures, but are typically not performed on an emergent basis. CT with contrast makes it difficult to visualize any obvious bleed. EEG is useful to evaluate for seizures, but this patient requires an urgent CT to evaluate for brain trauma.
51
Q

A 17-year-old man is brought to the emergency department following a history of fall from the bike while riding it without a helmet. On physical examination, his Glasgow coma scale (GCS) score is 3, and both pupils are bilaterally 7 mm, not reacting to light. A computed tomogram (CT) scan of the brain reveals diffuse brain edema with obliterated cisterns. He undergoes an emergency decompressive left hemicraniectomy. Postoperatively, he does not make any progress. He is found to have no doll’s eye response. The technologist is called to perform a nuclear medicine perfusion test. He places an elastic band around the patient’s head, just above the orbits. What is the rationale for this application?
1. This decreases the activity from the orbits.
2. This increases intracranial pressure, increasing the sensitivity of the exam.
3. This decreases the blood supply from small scalp veins.
4. This helps immobilize the patient’s head.

A

3. This decreases the blood supply from small scalp veins.

  • In a brain death study, there would not be any actual cerebral blood flow on the nuclear medicine perfusion test (NMPT).
  • One should not inadvertently confuse scalp veins with cerebral vessels and get a false negative study.
  • The elastic band minimizes scalp blood flow.
  • The apnea test remains the gold standard for final confirmation of brain death. NMPT is an ancillary test.
52
Q

A 42-year-old male with a past medical history of hypertension presents with disequilibrium and dysarthria lasting for nearly 12 hours. He has been coughing severely for the past 3 days due to an upper respiratory tract infection. His vital signs show a blood pressure of 140/90 mmHg, heart rate of 89/min, and a temperature of 37 C. On examination, the patient has a slurred speech with no facial asymmetry. The motor and the sensory exam were normal in all extremities. He was able to ambulate without assistance, however, would fall to his left when asked to do tandem gait. There was a slight overshoot on the left hand when doing the finger to nose test. CT angiogram shows a double-lumen appearance of the left vertebral artery with no evidence of thrombus formation. Which of the following is the most appropriate treatment for the patient?
1. Antiplatelet therapy
2. Thrombolytic therapy
3. Immediate surgery to repair the dissection
4. Supportive care only

A

1. Antiplatelet therapy

  • The goal of therapy is to prevent clot formation or treat clot formation. It is the thrombosis that causes stroke by clot embolization, not the dissection itself.
  • Start either antiplatelet and anticoagulant therapy once the diagnosis is made and no contraindications exist.
  • Supportive care is indicated in this patient but needs to be done in conjunction with antiplatelet and or anticoagulant therapy.
  • Thrombolytic therapy is a treatment option in patients with neurologic deficits presenting within 4.5 hours if they have no contraindications. This patient, however, presented 12 hours after the symptom onset. Surgical repair in this patient’s situation is not indicated. Most dissections heal spontaneously. The treatment is not aimed at the dissection but for thrombotic prevention. Surgery is reserved for those with recurrent dissections.
53
Q

A 70-year-old woman presents with gait instability. She says her balance is progressively getting worse. She would trip frequently and had two falls in the past month. All her vital signs are within normal limits. On examination, there is peripheral facial palsy on the left and decreased hearing. Weber test lateralizes to the right. She has mild dysmetria on the left hand. She veers to her left on walking, and she is unable to do tandem gait. Given the most likely diagnosis, which of the following is the most likely finding on MRI brain?
1. A characteristic “ice-cream cone appearance”
2. A homogeneously enhancing tumor with a dural tail
3. A ring-enhancing lesion on the pons
4. A hypointense lesion on T2 fluid-attenuated inversion recovery (FLAIR) image at the cerebellar peduncle

A

1. A characteristic “ice-cream cone appearance”

  • Acoustic neuroma produces the characteristic “ice-cream cone” appearance on contrast-enhanced imaging.
  • The signs and symptoms of acoustic neuroma are attributed to the involvement of the cranial nerve VIII, compressing surrounding cranial nerves, cerebellum, and brainstem.
  • The majority of acoustic neuromas present with unilateral hearing loss due to cochlear nerve interruption or impairment of blood supply to the nerve. Other clinical features include tinnitus, decreased word understanding, vertigo, headaches, and facial numbness.
  • Up to 70% to 90% of tumors in the cerebellopontine angle are acoustic neuromas.
54
Q

A 52-year-old woman presents to a clinic with dull low back pain, which first started after with soreness radiating to the bilateral buttock. The soreness is aggravated when asked to do the lumbar extension and rotation to the side. The Kemp test is also positive. X-ray of the spine shows mild L4-L5 disc space narrowing without spondylolisthesis. Improvement of pain with which of the following nerve blocks is most diagnostic of the patient’s underlying condition?
1. Medial branch of the posterior ramus
2. Lateral branch of the posterior ramus
3. Anterior branch of the posterior ramus
4. Posterior branch of the posterior ramus

A

1. Medial branch of the posterior ramus

  • The sign and symptoms in this patient are suggestive of facet arthritis. A medial branch block of the posterior ramus is diagnostic for facet arthritis.
  • The injection of corticosteroids or a local anesthetic into the facet joint is both diagnostic and therapeutic.
  • Some anesthetists suggest injecting corticosteroids at the site where the medial branch of the spinal nerve enters the bone. Fluoroscopy should be used to identify the notch between the superior articulating surface and the transverse process.
  • Great care should be taken not inject inside the vertebral artery or the radicular branches that enter the neural foramina.
55
Q

A 24-year-old male soldier is evacuated following a closed head injury from an improvised explosive device (IED). The patient is intubated and sedated with GCS 3T, and he has a large occipital hematoma. Vital signs are within normal limits. Based on the initial neurological exam, the initial evaluating clinician had a high suspicion of intracranial hemorrhage, but advanced imaging is not available at the current location. An optic nerve sheath ultrasound (ONSUS) is performed to evaluate intracranial hypertension. 3% hypertonic saline is infused. Which of the following pre- and post- optic nerve sheath diameter (ONSD) evaluations is most suggestive of intracranial hypertension and effectively instituted therapy in this patient?
1. ONSD of 6.2 mm pre-therapy and 4.8 mm post-therapy
2. ONSD of 6.8 mm pre-therapy and 6.4 mm post-therapy
3. ONSD of 4.8 mm pre-therapy and 5.2 mm post-therapy
4. ONSD of 5.2 mm pre-therapy and 5.2 mm post-therapy

A

1. ONSD of 6.2 mm pre-therapy and 4.8 mm post-therapy

  • As with most diagnostic imaging, trends and repeat evaluation are far more suggestive than a single number. An ONSD of 6.8 mm is extremely elevated, and a reduction to 4.8 mm would suggest decreasing ICP.
  • The ONSD changes in real-time based on ICP, and interventions to lower ICP can be monitored by serial exams.
  • Performing serial ONSUS is the only available modality in the described scenario. The US Army CPGs suggest an initial bolus of 250 ccs of 3% hypertonic saline followed by 50-100 mL/hr with target sodium of 150 to 160 mEq/L.
  • Conservative measures for improving cerebral perfusion include increasing systolic blood pressure (>110 mmHg) and elevation of the head of the bed to 30 to 45 degrees.
56
Q

A 56-year-old female without any medical history presents in the clinic with muscle weakness and easy bruisability. On examination, she has a round face with hair and purple striae on the abdomen. The patient also complains of having low mood and loss of energy for the past few weeks. She has a heart rate of 78 bpm and a blood pressure of 130/90 mmHg. What is the most common cause of the suspected diagnosis?
1. Adrenocortical adenoma
2. Pituitary microadenoma
3. Ectopic adrenocorticotropic hormone
4. Paraneoplastic syndrome

A

2. Pituitary microadenoma

  • Cushing disease is reserved for Cushing syndrome that is caused by excessive secretion of adrenocorticotropic hormone (ACTH) from a pituitary tumor, usually an adenoma.
  • Cushing disease is responsible for 70% of cases of endogenous Cushing syndrome. The rest of the cases are caused by ectopic secretion of ACTH by a tumor.
  • The pituitary tumors in Cushing disease are microadenomas, which measure 10 mm or less. These microadenomas do not cause mass effect and are often discovered after hypersecretion of ACTH is determined.
  • Macroadenomas are uncommon in patients with Cushing disease. These tumors cause mass effects such as suprasellar extension, optic chiasm compression, and cavernous sinus compression.
57
Q

A 47-year-old male complaining of difficulty seeing out of his left eye is brought to the emergency department. In the ED, the patient is noted to have 2/5 strength and diminished sensation on the right upper and lower extremities. The patient is additionally noted to have a left-sided drooping of the eyelid, and a left-sided pupil diameter of 1mm. After some time, the vision in his left eye has returned. Damage to which of the following structures has potentially led to the patient’s characteristic signs and symptoms?
1. Internal carotid artery
2. Ophthalmic artery
3. Vertebral artery
4. External carotid artery

A

1. Internal carotid artery

  • The patient in the question is experiencing a right-sided hemiparesis, sudden onset severe headache, ptosis (drooping of the left eyelid), miosis (1mm constriction of the left pupil), and transient monocular blindness (loss of vision in the left eye that has now returned. This is consistent with the internal carotid artery (ICA) dissection.
  • The risk factors for ICA dissection (and vertebral artery dissection) include sports trauma and minor trauma that can be experienced with chiropractic manipulation. Other factors which can increase the risk factor for ICA dissection include Ehlers- Danlos syndrome, fibromuscular dysplasia, and Marfan syndrome (all of which fall under connective tissue diseases). The connective tissue disorders make the layers of the blood vessel weaker and more prone to injury with lower trauma.
  • Patients with ICA dissections will primarily experience stroke- like or (TIA) Transient ischemia attack-like symptoms, though a number of patients will be asymptomatic. Those who are symptomatic can have hemiparesis, TIA which also includes transient monocular blindness, and Horner syndrome. Horner syndrome occurs because the sympathetic fibers of the superior cervical ganglion partly travel with the internal carotid artery. Dissection of the ICA can lead to distention of these fibers and cause Horner syndrome. In this case, anhidrosis was not seen but miosis and ptosis were.
  • ICA dissection and vertebral artery dissection are mostly similar and have similar clinical presentations as well as the same risk factors. The common carotid artery gives off no branches beyond the two terminal branches, ICA, and ECA. The external carotid artery provides blood supply to many structures in the face, neck, and skull but is generally not thought to supply the brain parenchyma whereas the ICA does not have many branches until intracranial at which point it gives off the Middle cerebral artery and anterior cerebral artery.
58
Q

A 17-year-old male presents with severe headaches. He emigrated from Mexico 5 months ago. He noted headaches about a month ago, on the frontal region, initially graded 3/10, and gradually progressed to 7/10. This is more prominent in the mornings and accompanied by nausea. He also claims that for the past week, he has been having vivid visual hallucinations of animals like goats and insects in his room, which he can describe in great detail. On examination, BP 140/90 mmHg, HR 54 beats/min, RR 18 breaths/min, T 99.6 F. His pupils are isocoric, briskly reactive to light, there are no visual field cuts, and fundoscopy reveals bilateral papilledema. There is no facial asymmetry or dysarthria. The motor and sensory examinations are normal. There is positive nuchal rigidity, Brudzinski, and bilateral Babinski. The head CT was normal. The chest x-ray shows pleural apical thickening.The CSF examination reveals: an opening pressure of 23 cm/H2O, WBC 345/mm^3, 85% lymphocytes, glucose 23 mg/dL, and protein 90 mg/dL. Which of the following etiologic agents is most likely?
1. Mycobacterium tuberculosis
2. Cryptococcus neoformans
3. Herpes simplex
4. Rickettsia rickettsia

A

1. Mycobacterium tuberculosis

  • Tuberculous meningitis (TBM) is a manifestation of extrapulmonary tuberculosis caused by the seeding of the meninges with the bacilli of Mycobacterium tuberculosis (MTB).
  • Tuberculous meningitis presents 1% of all cases of extra- pulmonary TB. In the developed world where there is a lower prevalence of TB in the population, estimates are that TBM accounts for 6% of all causes of meningitis.
  • In locations with a higher prevalence of MTB in the population, estimates are that TBM accounts for up to one third to one half of all bacterial meningitis.
  • Tuberculous meningitis assessment is by obtaining cerebrospinal fluid (CSF) for analysis. Typically, the CSF reveals low glucose, elevated protein, and modestly elevated WBC count with a lymphocytic predominance. The CSF analysis most closely resembles the CSF analysis of viral meningitis.
59
Q

A 50-year-old woman presents to the emergency department after sustaining an injury to her head and neck while diving into the swimming pool. Her blood pressure is 130/80 mmHg, heart rate is 85 beats/min, arterial oxygen saturation is 99% on room air, and her body mass index (BMI) is 18 kg/m2. A cervical orthosis has been placed by the emergency medical services (EMS) team. The patient is alert and oriented to person, place, time, and situation. Her physical examination reveals tenderness to palpation over the upper cervical spine and severe soft tissue damage over the eyebrows bilaterally; otherwise, the rest of the musculoskeletal examination is nonsignificant. Her CT scan of the head and cervical spine is in plan. What is the most appropriate step in the management of her cervical spine?
1. Definitive treatment with halo vest immobilization
2. Remove the cervical orthosis and no immobilization is needed
3. Continue with cervical orthosis until it is determined if planned surgical intervention is needed
4. Immediate surgical intervention with post-operative adjuvant halo vest immobilization

A

3. Continue with cervical orthosis until it is determined if planned surgical intervention is needed

  • She has most likely C1 vertebral fracture. These are an indication for definitive treatment with halo vest immobilization (HVI); however, severe soft tissue disruptions over the proposed pin sites are absolute contraindications. This patient has severe soft tissue disruption over where the anterior pins would be placed. Therefore this is contraindicated in this patient.
  • Immobilization is necessary when a trauma patient presents with a tender cervical spine. Furthermore, CT imaging confirms a cervical spine fracture; therefore, it is required to continue with cervical immobilization with a cervical orthosis.
  • This patient has an upper cervical spine fracture. Definitive treatment requires immobilization, which can be achieved with either surgical intervention or rigid external immobilization via the halo vest immobilizer. Halo vest immobilization (HVI) is contraindicated in this patient; therefore, surgical immobilization is the most appropriate treatment plan for this patient if indicated. Since the patient has an unstable cervical spine injury, they need to maintain a cervical collar until the planned procedure.
  • Surgery is an appropriate option, and HVI can be placed for temporary postoperative adjuvant immobilization in these injuries; however, this patient has an absolute contraindication because of severe soft tissue disruption where the anterior pin has to be placed. Therefore this is incorrect.
60
Q

A 25-year-old male basketball player presented to the clinic with right-sided low back pain described as shooting in quality and graded as 9/10 in intensity. This was experienced during his training and matches. It began some time ago as discomfort that responded to NSAIDs and rest. The patient at the time did not seek medical advice and continued playing basketball, and the pain increased. There are now paresthesias and mild weakness in the dorsiflexion of the right foot. Fracture at the right L5 pars interarticularis is suspected. At what point does this problem usually present with pain?
1. During early teenage years when the slippage is 15%
2. During early teenage years when the slippage is 25%
3. During the late teenage years the to early 20s when the slippage is 25%
4. When the patient is over 60 years and the slippage is 50%

A

3. During the late teenage years the to early 20s when the slippage is 25%

  • Spondylolysis is a problem from the late teenage years to the early 20s. A positive stork test that is single leg hyperextension and rotation of the spine, which reproduces the patient pain, is diagnostic of spondylolysis until proven otherwise.
  • It becomes symptomatic when there is at least 25% slippage.
  • Some patients have no risk factors, but recurrent lumbar hyperextension does predispose people to this condition.
  • Conservative treatment usually suffices, but if cord compromise occurs, surgery is necessary.
61
Q

A 43-year-old man was brought in by paramedics after a motor vehicle accident. He was unconscious at the scene. On examination, the patient opens his eyes to pain stimulation, is incomprehensible, and withdraws his extremities to pain stimulation. Cranial nerve examination show isocoric pupils, briskly reactive to light, primary gaze midline, and no facial asymmetry. He withdraws all extremities to pain stimulation. Babinski’s sign is positive bilaterally. A head CT scan is unremarkable. Which of the following is the pathophysiology of the patient’s current presentation?
1. Disconnection or malfunction of the interconnection of neurons
2. Biochemically induced neuronal dysfunction
3. Disturbance of cellular metabolism and accumulation of intracellular water
4. Damaged neurons due to intracellular energy failure and cell death

A

1. Disconnection or malfunction of the interconnection of neurons

  • Diffuse axonal injury (DAI) is a disruption of the cytoskeleton of the axonal portion of a neuron. There is damage or disruption of the interconnection of neurons.
  • This is commonly caused by traumatic acceleration and deceleration injury. DAI is a clinical diagnosis. Typically, DAI is considered in patients with a Glasgow coma scale (GCS) of less than 8 for over six hours.
  • A head CT has a low yield in detecting diffuse axonal injury related injuries, but severe forms may show small punctate hemorrhages to white matter tracts. Currently, magnetic resonance imaging (MRI), specifically diffuse tensor imaging (DTI), is the imaging modality of choice for the diagnosis of diffuse axonal injury.
  • Concussion causes biochemically induced neuronal dysfunction. In cytotoxic cerebral edema, there is a disturbance of cellular metabolism and accumulation of intracellular water. In hypoxia, the neurons are damaged due to intracellular energy failure and lead to cell death.
62
Q

A 67-year-old man with a history of bitemporal hemianopsia is scheduled for surgery. Magnetic resonance imaging of the head and neck reveals a craniopharyngioma. Transsphenoidal hypophysectomy is performed without apparent complications. Two hours after surgery, he develops polyuria (375 mL/h) and hypernatremia (155 mEq/L). The patient describes severe thirst and seems lethargic and disoriented. Which of the following is the next best step in the management of this patient?
1. IV furosemide
2. Intravenous 5% dextrose and water
3. Intravenous desmopressin
4. IV mannitol

A

3. Intravenous desmopressin

  • Diabetes insipidus is a common complication following pituitary surgery. Diabetes insipidus (DI) is a disease process that results in either decreased release of or response to antidiuretic hormone (ADH, also known as vasopressin or AVP), which can cause electrolyte imbalances.
  • Most cases of diabetes insipidus following transsphenoidal hypophysectomy are transient. About 20% of patients who undergo neurosurgery will present with diabetes insipidus
  • Treatment is with fluid replacement and desmopressin. The most common findings in patients with diabetes insipidus are polydipsia, polyuria, and nocturia. IV D5W can be used for IV fluid replacement, but IV desmopressin is the better choice. The other choices are not effective for diabetes insipidus.
  • Risk factors for developing diabetes insipidus due to transsphenoidal hypophysectomy include craniopharyngioma or extensive intraoperative handling of the pituitary gland
63
Q

A 55-year-old male highway worker from Kentucky is admitted via emergency with complaints of new-onset seizures and fever. He was suffering from a fever and cough for the last few days and complained of a new-onset headache for the last two days. Today he developed jerky movements of his face and arm followed by loss of consciousness and jerking of all 4 limbs, which lasted for 3 minutes. The event was witnessed by his daughter, who contacted the emergency services. He was transferred to the acute medical unit and is currently stable. His vitals are a temperature of 100 F, pulse of 95 beats per minute, Respiratory rate of 25 breaths per minute, and blood pressure of 100/50 mmHg. He still complains of headache, and his neurological assessment shows decreased strength in the right upper and lowers limb and mild facial deviation towards the left side. Auscultation of his chest reveals coarse crackles in the left lower lobe. His cardiovascular and gastrointestinal examination is unremarkable. He is started on broad-spectrum antibiotics and anticonvulsants. His x-ray reveals a consolidation with reticulonodular shadowing in his left lower region. A CT scan with contrast shows the presence of a solitary lesion in the frontoparietal area with peripheral enhancement and significant perilesional edema. A brain biopsy is arranged which demonstrates necrotizing granulomas with the presence of 10 microns sized pathogens, with thick wall and broad-based budding on Grocott- methenamine silver stain. What medication should be added to the patient’s current treatment regimen?
1. Oral itraconazole
2. Antituberculosis therapy
3. Sulphadiazine-pyrimethamine
4. Amphotericin B

A

4. Amphotericin B

  • This patient likely has a disease of infectious etiology involving his lungs and brain. The presentation, along with his occupation and biopsy findings, makes the likely diagnosis to be disseminated blastomycosis. The recommended treatment for moderate to severe blastomycosis is amphotericin B.
  • Amphotericin B is a broad-spectrum antifungal that exerts its fungicidal action by binding to ergosterol, a key component of fungal cell membranes. It is the recommended treatment for life- threatening blastomycosis and also in those with CNS disease. It has an extensive side effect profile and can cause renal dysfunction, hypokalemia, anemia, and thrombophlebitis, which can limit its use.
  • The recommended treatment dose of amphotericin in blastomycosis is 0.7 to 1 mg per day to a total dose of 1.5 to 2 gm. It has been seen that higher total doses are associated with a decreased frequency of relapse.
  • The liposomal formulation of amphotericin B is associated with less toxicity and is preferred where available. Oral itraconazole has poor CNS penetration and is avoided in severe disease. Voriconazole may be an alternative to amphotericin B. Sulphadiazine-pyrimethamine is recommended in toxoplasmosis, whereas antituberculous therapy is effective in treating tuberculosis.
64
Q

A 53-year-old man presents to the clinic, continuing to have neck pain that radiates to his left arm. MRI spine done four months ago showed early disc desiccation at C5-6 disc space. Two months ago, he underwent placement of a permanent cardiac pacemaker following a third-degree atrioventricular block. His neck pain has now affected his activities of daily living as well. Which of the following is the most rational approach for diagnosing the clinical entity in the patient?
1. Repeat MRI cervical spine
2. Cervical provocation discography
3. Ultrasound spine
4. Nerve conduction velocity (NCV) study

A

2. Cervical provocation discography

  • The most important adjunct in the evaluation of cervical discogenic pain syndrome is cervical provocation discography.
  • Cervical provocation discography is an image-guided procedure where a contrast agent is injected into the nucleus pulposus of the suspected intervertebral disc. The study includes disc stimulation and morphological assessment; it not only identifies painful cervical intervertebral disc, but it can depict internal derangements as well.
  • In cervical discogenic pain syndrome, radiographic findings may be normal. It may show degenerative changes, but there is little correlation with symptomatic vs. asymptomatic individuals based solely on these findings.
  • Dynamic MRI cervical spine is indicated in suspected cases of Hirayama disease. MRI is contraindicated in patients with a cardiac pacemaker.
65
Q

Which is true regarding MR imaging of dural sinus thrombosis?
1. MRI easily demonstrates acute thrombosis
2. MRI easily demonstrates subacute thrombus
3. Time-of-flight (TOF) magnetic resonance venography (MRV) shows subacute thrombus
4. Contrast-enhanced MRV shows chronic thrombus

A

2. MRI easily demonstrates subacute thrombus

  • MRI has difficulty showing acute thrombosis because of the presence of deoxyhemoglobin. This makes the thrombus isointense on T1 and hypointense on T2, indistinguishable from normal flow void.
  • Clot appears bright on both T1 and T2 MRI of subacute thrombus.
  • TOF MRV is poor in subacute situations because the normal T1 hyperintensity falsely simulates normal flow of a patent sinus.
  • Contrast-enhanced MRV is poor in chronic situations because chronic thrombus can enhance secondary to organizing fibrosis, and thus falsely simulate normal flow of a patent sinus.
66
Q

A 45-year-old man has been brought to the emergency department after being struck on the head by a beam while working in a factory. He is alert and oriented in time and place, opens his eyes spontaneously, and is moving all extremities; however, he does not remember the events leading up to his presentation. His physical examination reveals an obvious depression deformity on the forehead with an overlying laceration. He denies any past medical history and has not been vaccinated in 15 years. In addition to completing an advanced trauma life support (ATLS) driven primary survey and obtaining a computed tomography scan of the head and facial bones, what is the next best step in the management of this patient with the injury above?
1. Antibiotics and Tdap (tetanus, diphtheria, pertussis) immunization
2. Laceration repair
3. Rapid sequence intubation
4. Immediate neurosurgery consultation

A

1. Antibiotics and Tdap (tetanus, diphtheria, pertussis) immunization

  • This patient has an open frontal sinus fracture requiring antibiotics, and a tetanus toxoid shot since the patient has not received a booster within the last ten years. Amoxicillin with clavulanic acid is the initial antibiotic of choice if there is no intracranial extension of the injury. If there is a displaced posterior table fracture, a third-generation cephalosporin should be administered for coverage across the blood-brain barrier.
  • The sinuses are considered sterile under normal conditions, but the introduction of debris and skin flora is possible after such an injury and requires the appropriate systemic antibiotics coverage. Coverage for both Staphylococcus aureus and methicillin-resistant Staphylococcus aureus (MRSA) should be considered in these patients.
  • This condition, if untreated, could lead to complications like meningitis, frontal sinusitis, or osteomyelitis.
  • In the case of a frontal sinus fracture, appropriate antibiotic coverage is needed, and a Tdap booster is also required to cover for Clostridium tetani, which can lead to respiratory distress and death. The wound should be decontaminated, followed by closure; however, this is not sufficient to prevent local infection and possibly tetanus. Neurosurgery consultation may be indicated depending on the underlying injuries; however, based on the initial assessment of this patient, a computed tomography scan of the head and facial bones should be obtained prior to the consultation.
67
Q

A 69-year-old male with a previous history of transsphenoidal surgery for the management of Cushing disease five years prior is evaluated by the endocrinologist and the general surgeon for treatment failure. He is now being considered for bilateral adrenalectomy, as he is not controlled with medical therapy. Which postadrenalectomy factor in this patient is most predictive for developing a new pituitary tumor?
1. High levels of plasma adrenocorticotropic hormone
2. High levels of plasma cortisol
3. Old age
4. Male gender

A

1. High levels of plasma adrenocorticotropic hormone

  • Risk factors for developing Nelson syndrome have been studied with means of achieving early detection and treatment. High levels of plasma adrenocorticotropic hormone following bilateral adrenalectomy have the strongest predictive capabilities.
  • Most of the Nelson syndrome cases are seen as a result of bilateral adrenalectomy for the treatment of Cushing disease.
  • It is hypothesized that the loss of feedback inhibition of the hypothalamic-pituitary-adrenal axis leads to the development of an adrenocorticotropic hormone-secreting pituitary tumor.
  • Age and sex have not been significantly associated with increased risk; however, younger patients undergoing bilateral adrenalectomy have a higher lifetime risk of developing Nelson syndrome.
68
Q

A 34-year-old male presents to the emergency department after a motor vehicle collision. His neurological exam reveals 3+ strength in all muscle groups in his lower extremities, no altered sensation, bulbocavernosus reflex is present. CT scan of the thoracolumbar spine reveals a T12 burst fracture with moderate to severe spinal stenosis. Subsequent MRI demonstrates edema/tearing to the posterior ligamentous complex (PLC) at T12- L1. What would be his Thoracolumbar Injury Classification System Score (TLICS), and recommended treatment?
1. TLICS Score of 8; TLSO brace, IV steroids, monitor for improvement
2. TLICS Score of 7; Posterior surgical decompression and stabilization
3. TLICS Score of 5; Bedrest and immobilization
4. TLICS Score of 8; Posterior surgical decompression and stabilization

A

4. TLICS Score of 8; Posterior surgical decompression and stabilization

  • This patient’s Thoracolumbar Injury Classification System (TLICS) score adds to 8, (morphology = 1 + 1 for burst fracture = 2, incomplete spinal cord injury = 3, posterior ligament complex injury/tear = 3). The most appropriate treatment would be surgical decompression from the burst fracture through a posterior approach and stabilization. Stabilization is warranted due to the tearing of the PLC with anterior stabilization. This correlates to a 3 column injury, and the spine is inherently unstable.
  • Thoracolumbar Injury Classification System (TLICS) categorizes injury by morphology (compression, burst, translation/rotation, distraction), posterior ligamentous complex integrity (intact, indeterminate, disrupted) and neurological status (intact, complete, incomplete, cauda equina). The TLICS scoring is a tool used during thoracolumbar trauma and can guide treatment. Scores of 5 or greater are recommended for surgery.
  • The TLICS score is unique in that it places a significant emphasis on the integrity of the PLC. The PLC consists of the supraspinous ligament, interspinous ligament, ligamentum flavum, and facet joint capsules. Disruption of the PLC is typically indicated by widening of the interspinous space or of the facet joints, empty facet joints, facet perch or subluxation, and dislocation of the spine. MRI is the best imaging study to examine the integrity of the PLC and is indicated for any burst fracture or spinal column fracture with neurologic deficits to determine the extent of soft tissue injury and neurologic injury.
  • Bracing, steroids, or bedrest would be improper management due to the patient’s abnormal neurologic exam and extent of injury seen on CT and MRI. Time to the operation room and decompression/stabilization will give this patient the best chance for recovery and to decrease morbidity.
69
Q

A 9-year-old boy is brought to the hospital following a fall from a two-storied building. The neurological examination reveals paraparesis in the child with the American spinal injury association (ASIA) grade ‘C’ status. The MRI spine reveals a burst fracture of the T8 vertebra with Meyerding grade 2 subluxation causing cord contusions. A short-segment implant fixation of the traumatic spine is planned from the posterior approach. Which of the following complications is most likely to occur with this procedure in this patient?
1. Dural injury
2. Vascular injury
3. Pedicle violation
4. Implant failure

A

3. Pedicle violation

  • The pedicle screw fixation is the most common method of the spinal fixation since it incorporates all the three Denis columns of the spine thereby maximizing its stability.
  • The pedicle screw placement in the children is, however, challenging owing to its anatomical variability compared to their adult counterparts. The pedicles are of smaller width, and the spinal canal is comparatively narrower.
  • There is a high risk of pedicle violations during the procedure. The estimated risk of either the medial or the lateral violation of the pedicle screw in the pediatric population is approximately 16% in the postoperative CT images.
  • The incidence of vascular injury and the nonunion following the implant fixation in pediatric patients are minimal compared to that of the risk of screw malpositions.
70
Q

A 33-year-old female active recreational volleyball player presents to the clinic with severe axial spine pain right worse than left with intermittent right leg pain in a mixed dermatomal distribution. She felt a sharp pain suddenly when diving for a ball on a sand volleyball court 6 weeks ago. She continued to play that day and developed severe pain over the next 72 hours, and has not done any sports activities since. She reports components of pain and numbness. She had pain once in a while on the right playing in college, but “it’s never been anything like this.” She denies any loss of bowel and or bladder control. Gait has a decreased stance phase on the right. Muscle stretch reflexes are preserved to the patellar, medial hamstring, and Achilles. Seated slump testing and straight leg raise are positive on the right with some cross-referred pain to the right when straight leg raising the left. She has decreased light touch and pinprick to the ASIA (American Spinal Cord Injury Association) sensory points on the right. Her beta-HCG is negative. Plain films of the lumbosacral spine show some loss of the height of L4/5 and L5/S1 with some loss of endplate definition at L5/S1L4/5 with no anterolisthesis or retrolisthesis. The spine is stable on flexion/extension. MRI LS spine shows subligamentous paramedian bulges at L4/5 and L5/S1 without extruded fragments and some mild facet hypertrophy. She has had a lumbar epidural corticosteroid injection that helped with the leg pain for about a week. She has been offered a rather extensive spinal decompression/fusion, but she just got married this past year, wants to start a family, and makes it clear she absolutely does not want to go in that direction if there is any other option. Which of the following best describes the rationale of provocative discography with post discogram CT for this patient?
1. Assessment of the outer annulus not seen on MRI
2. Assessment of which levels to fuse during surgery
3. Ruling out sacroiliac or facet joints as the cause of her pain
4. Assessment of endplate driven changes

A

1. Assessment of the outer annulus not seen on MRI

  • A main point of discography is a real-time evaluation of the internal morphology of the disk and CT axial scanning for outer annular tears that are of clinical significance. This is of value in planning intradiscal treatments such as PRP, regenerative agent injections, intradiscal thermal treatments, and/or endoscopic diskectomy.
  • Discography may have a role in surgical planning but is just one part of a workup that is done within the context of clinical exam, history, comparison to other imaging studies, usually also with an EMG. Grossly abnormal grade 5 disks leaking opening anteriorly and posteriorly may potentially be painless despite being unstable and appropriate for fusion.
  • Discography has nothing to do with SIJ and facet-mediated pain. She may have some degree of SIJ pain independent of the discogenic pain. Something simple as altered gait mechanics may cause SIJ pain and or dysfunction.
  • Endplate-driven changes are not differentiated from annular- driven changes but may be apparent even on the fluoro images at the time of the discogram. Type 1 vs 2 Modic endplate changes are differentiated on MRI.
71
Q

A 35-year-old woman presents due to double vision and left facial droop. Her symptoms started about a week ago. She says she noticed double vision when looking to the left and drooling from the left side of her mouth. Her past medical history includes migraines and multiple sclerosis. On examination, there is an impaired lateral gaze of the left eye. She cannot raise her eyebrow, close her eyes tightly, or raise her lips when asked to smile on the left. The motor exam shows a drift on the right upper extremity. Babinski sign is positive bilaterally. What is the most appropriate next step in the management of this patient’s condition?
1. Cranial CT with contrast
2. Cranial MRI with contrast
3. CT angiogram of the head
4. Lumbar puncture

A

2. Cranial MRI with contrast

  • Millard-Gubler syndrome (MGS) is one of the classical crossed brainstem syndromes characterized by a unilateral lesion of the basal part of the caudal pons.
  • The 6th and 7th cranial nerves and the corticospinal tract are involved in this syndrome.
  • MRI of the brain is more sensitive and specific than CT scan to small pontine lesions. Also, in a patient with multiple sclerosis (MS), MRI with contrast can delineate between new and old lesions.
  • Causes of MGS include tumors, infections, MS, and vascular- related lesions.
72
Q

A 24-year-old man comes to the office for the evaluation of a seizure episode. He describes a single episode of generalized seizures two days ago. He was experiencing a mild headache with nausea and vomiting for the last five weeks. The examination shows optic disc swelling. Magnetic resonance imaging (MRI) of the brain reveals a well defined and hypointense mass. Appropriate treatment is recommended. Which of the following evaluation is required after treatment in this patient?
1. MRI in the first 24-72 hours
2. Computed tomography (CT) scan in the first 12 hours
3. Positron emission tomography (PET) scan
4. Biopsy

A

1. MRI in the first 24-72 hours

  • This patient has brain cancer. In 20-40 years of age, low-grade gliomas are common.
  • MRI defines low-grade glioma (LGG) features and surgery is the recommended treatment in LGG.
  • Postoperatively, to plan a subsequent therapy, MRI is performed in the first 24-72 hours. Observation is recommended to detect any neurologic deficits afterward.
  • Postoperative evaluation is required in brain tumors, and MRI is the best modality. CT, PET, and biopsy can help in diagnosis but are not required for follow-up of brain tumors.
73
Q

A 68-year-old man presents to the clinic with pain in his low back and buttocks for the past 5 months. He has a history of hyperlipidemia and hypertension, for which he takes atorvastatin and amlodipine. The patient reports he works as a teacher and that the pain is worse when he is standing up to write on the board during lectures but improves when he sits down. The patient denies any trauma to the area. On physical exam, pain is elicited with lumbar extension. The sensation is intact to light touch throughout, and strength is 5/5 in bilateral lower extremities. On EMG, fibrillations and positive sharp waves at different levels bilaterally are seen. Which of the following findings is most likely to be seen on a spinal MRI in this patient?
1. Anteroposterior diameter of the lumbar spinal canal 18 mm
2. Anteroposterior diameter of the lumbar spinal canal 15 mm
3. Anteroposterior diameter of the cervical spinal canal 15 mm
4. Anteroposterior diameter of the lumbar spinal canal 10 mm

A

4. Anteroposterior diameter of the lumbar spinal canal 10 mm

  • The average diameter of a lumbar spinal cord is usually greater than or equal to 15 mm.
  • When there is a diameter noted to be less than 12 mm, this is considered significant for spinal stenosis.
  • The average diameter of a cervical spinal cord is usually greater than or equal to 13 mm. When there is a diameter noted to be less than 10 mm, this is considered significant for cervical spinal stenosis.
  • Anything less than 12 mm is considered significant for spinal stenosis. This would best fit the patient described in the question stem. Statin-induced myopathy and peripheral neuropathy should also be in the differential diagnosis, along with vascular claudication.
74
Q

A 56-year-old male construction worker presents to the clinic with severe back pain for more than ten years. He had a fall from a second-floor and fractured his L4 vertebrae 17 years ago, which was treated with a body jacket. Neurological examination shows intact motor and sensory function. Studies show severe L4-L5 degenerative changes with 20% anterolisthesis, which progress to 35% with flexion. He undergoes transpsoas lumbar interbody fusion L4-L5. Upon waking from anesthesia, he is found to have weakness in the quadriceps with power 3/5. When is the most appropriate time to perform the electromyography (EMG) study to confirm if the injury is neurapraxic?
1. 1-3 days after injury
2. 3-6 days after injury
3. 7-14 days after injury
4. 14-21 days after injury

A

4. 14-21 days after injury

  • EMG is most useful 2 to 3-weeks after the injury as the involved muscles had undergone denervation. EMG is done every six weeks. Stimulation of the nerve below the lesion will result in a compound muscle action potential (CMAP) wave from the neurapraxic axons.
  • Needle EMG is the most sensitive study. The motor response amplitude decrement begins around days 2–3 and is complete by day 6. After a 2- to a 3-week interval, the involved muscles undergo denervation, and EMG is indicated.
  • After a 2 to 3-week interval, when small-amplitude, if short- duration (SASD) motor unit potentials are detected in the EMG, the prognosis for recovery is excellent.
  • In a complete neurapraxia lesion, no motor unit action potentials (MUAPs) are elicited under voluntary control. EMG can differentiate if the cause of weakness is neuropathic or myopathic. In neurapraxic lesions, the muscle does not reveal any abnormal spontaneous activity (fibrillation and positive sharp waves). On follow-up, the presence of MUAPs indicates that reinnervation is taking place.
75
Q

A 70-year-old man with a history of prostate cancer presents to the clinic for progressive upper back pain with leg weakness. On examination, he has 2/5 strength in bilateral lower extremities, decreased sensation below T8 dermatome, and patellar reflexes are +3. MRI and CT imaging demonstrate a tumor involving the T8 vertebral body, bilateral pedicles, and canal compromise most consistent with prostate metastases. His oncologist deems he could have a good prognosis with surgery, and he had good function prior to his neurologic decline. The patient is being seen in a smaller community hospital without thoracic surgery availability, and the patient refuses to transfer hospitals. What is the most appropriate management strategy for this patient?
1. Do not offer surgery due to advanced weakness and lack of resources
2. Recommend radiation therapy alone to shrink the tumor
3. Offer mini-open lateral intracavitary corpectomy and debulking
4. Offer posterior laminectomy, costotransversectomy, bilateral pediculotomy with posterolateral corpectomy with adjuvant posterior pedicle screw fixation

A

4. Offer posterior laminectomy, costotransversectomy, bilateral pediculotomy with posterolateral corpectomy with adjuvant posterior pedicle screw fixation

  • Given the patient’s significant disability and that he was functional otherwise with good oncologic prognosis, surgery must be considered.
  • Surgery is much preferred over radiation therapy alone as he already has significant cord impingement with weakness to his lower extremities. Direct surgical debulking is preferred. Adjuvant postoperative radiation may be considered in most cases, but surgery is the mainstay here.
  • Only offer surgeries that you feel comfortable performing, and your hospital can accommodate. Lateral approaches to the thoracic spine are viable options, but if there is no thoracic surgery on standby, it will put the patient at extra risk in case of complications.
  • In addition, posterior pedicle subtraction with corpectomy through a costotransversectomy provides excellent visualization and access to both sides of the vertebra, which a lateral approach does not afford so easily. Posterior fusion will need to be supplemented with T8 graft placement.
76
Q

A 65-year-old patient is brought to the clinic with complaints of recent onset, slowly progressive hearing loss. He also has difficulty walking and has a tendency to fall towards his left. On examination, he has a sensorineural pattern of hearing loss. He also has nystagmus and dysdiadochokinesia. Which of the following is the best diagnostic modality in further evaluation of this patient?
1. Otoscopy
2. Pure tone audiometry
3. MRI brain
4. CT temporal bone

A

3. MRI brain

  • The patient has sensorineural hearing loss with cerebellar signs that suggest lesion originating in the internal auditory meatus and compressing upon the cerebellum.
  • Acoustic neuroma that grows into the cerebellopontine cisterns typically presents with such signs and symptoms.
  • MRI brain allows for better stratification of the management plan and grading of such lesions. With newer techniques such as tractography and constructive interference steady state (CISS), MRI also helps in forming surgical strategy in safe excision of the lesion after safeguarding facial nerve.
  • Otoscopy and CT temporal bone help in the assessment of patients with conductive pattern hearing loss. PTA only helps in determining the pattern and severity of hearing loss.
77
Q

A 45-year-old male patient with a history of alcohol abuse falls down a flight of stairs. A friend rushed him to the emergency department. Immediate evaluation with non-contrast CT of the head reveals a subdural hematoma which required surgical intervention. Currently, the patient is stable, but he is having significant difficulty with memory. He can recognize his family members and talk about his childhood and tell stories of when he was in the army ten years ago but has difficulty with remembering what he did earlier that day or why he is in the hospital. What part of the brain is most likely injured?
1. Cerebellum
2. Occipital lobe
3. Parietal lobe
4. Prefrontal cortex

A

4. Prefrontal cortex

  • The prefrontal cortex is involved in short-term memory.
  • It must cooperate with other parts of the cortex as it extracts information for brief periods.
  • Short-term memory has a limited capacity.
  • Transfer of information to long-term memory is needed for more permanent storage.
78
Q

A 73-year-old female complains of severe back pain for several years. She is found to have a moderate sagittal imbalance of 8 cm on lateral spine radiographs. Physical therapy has not given her meaningful relief, and she plans to undergo an osteotomy procedure to correct her deformity. Her surgeon determines that she has adequate disk heights and mobility, and desires an osteotomy that would provide up to 10 degrees of correction of her sagittal imbalance that would not require anterior fusion at the level of the osteotomy. Which of the following procedure would be most appropriate for this patient?
1. Pedicle subtraction osteotomy
2. Salter pelvic osteotomy
3. Smith-Petersen osteotomy
4. Vertebral column resection

A

3. Smith-Petersen osteotomy

  • A Smith-Petersen osteotomy is indicated in cases of mild- moderate sagittal imbalances.
  • Smith-Petersen osteotomies provide up to 10 degrees of correction per level of the osteotomy.
  • No anterior fusion is required when performing a Smith- Petersen osteotomy.
  • Pedicle subtraction osteotomies and vertebral column resection are indicated in cases of severe sagittal imbalance and provide much more correction than a Smith-Petersen osteotomy. Salter osteotomies of the pelvic are pediatric procedures used in the setting of dysplastic hips.
79
Q

A 49-year-old patient presents with difficulty in maintaining balance. He noted his symptoms about a month ago, he claims of frequent tripping and a generalized feeling of instability. He was diagnosed with diabetes 1 year ago. He is a smoker and drinks 6-8 bottles of beer daily. He had 3 sexual partners in the past month and admitted to not using barrier protection. On examination, he was awake, alert, able to follow commands, Pupils were small, isocoric, with poor constriction to light bilaterally, (+) constriction on accommodation. The extraocular muscle movements were intact, and there is no facial asymmetry noted. The motor examination shows 5/5 on all extremities. The sensory exam shows decrease position and vibration sense on both lower extremities up to midcalf. Sensation to light touch was intact. Which of the following conditions is most likely to cause this patient’s symptoms?
1. Chronic ethanol abuse
2. Multiple sclerosis
3. Syphilis
4. Diabetes mellitus

A

3. Syphilis

  • Argyll-Robertson pupils is highly specific for late-stage neurosyphilis.
  • Argyll-Robertson pupils are irregular miotic pupils. There is no pupillary light response, but there is brisk pupillary constriction to near stimuli. Dilatation in the dark is poor.
  • The anterior visual pathway function is normal. It is most commonly caused by a lesion in the region of the Sylvian aqueduct in the rostral midbrain.
  • This interferes with supranuclear inhibitory fibers and fibers for light reflex close to the Edinger-Westphal nuclei.
80
Q

A 28-year-old G1P1 woman presents to the clinic for postpartum follow-up after delivering her first child four days ago. The delivery was uncomplicated, but her infant was almost 10 pounds (4.5 kg) at birth. She has no significant past medical history, but she has been complaining of low back pain, some left anterior leg numbness, and weakness going from sitting to standing. This pain started after her epidural wore off following delivery. Her pain has not gotten any better or worse; it is constant and burning in nature. She denies any bowel or bladder incontinence. On physical exam, she is noted to have diminished sensation along the L4 dermatome of her left leg. Lower extremity muscle strength is 3/5 on the left, 5/5 on the right. Her deep tendon reflexes are 2/4 for L4, S1. The straight leg raise is negative. Which of the following is the next best step in the management of this patient?
1. Urgent neurosurgical evaluation
2. Ibuprofen
3. Physical therapy
4. MRI of the lumbar spine

A

2. Ibuprofen

  • Lumbosacral plexopathy can be a transient disorder, as in the case of pregnancy.
  • Symptoms most often occur between 32-34 weeks gestation.
  • Postpartum lumbosacral plexus symptoms arise due to birth trauma and direct compression caused by the fetal head at the pelvic brim.
  • The majority of patients have a complete resolution of their symptoms two to six months following delivery.
81
Q

A 47-year-old man presents to the orthopedics department with a 3-week history of fever, progressive back pain, and difficulty with ambulation. He is taking ibuprofen and acetaminophen over the counter with no relief. Which of the following radiologic tests has the highest sensitivity and specificity for the condition in this patient?
1. Plain radiograph of the spine
2. Nuclear medicine scan with gallium-67 isotope
3. Computed tomography (CT) scanning with the administration of intravenous contrast
4. Magnetic resonance imaging (MRI) with the administration of intravenous gadolinium

A

4. Magnetic resonance imaging (MRI) with the administration of intravenous gadolinium

  • Discitis is an infection of the intervertebral disc space. The role of the intervertebral discs is to separate and cushion the spinal segments from each other. An infection, and thus inflammation of these discs can cause much pain and discomfort. MRI is the best test for this patient.
  • MRI is the most sensitive and specific test for diagnosis.
  • MRI will help the physician diagnose discitis at an early stage and also helps diagnose other causes of back pain (eg, herniated intervertebral disk, spinal stenosis).
  • Treatment options range from immediate broad-spectrum antibiotics to awaiting cultures for sensitivity before administering antibiotics, to spinal bracing alone without antibiotics.
82
Q

A 55-year-old man patient from Japan presents with a four-month history of weakness in his legs. He is now struggling to get out of bed unaided and also reports urinary incontinence. His past medical history includes a kidney transplant from his brother three years ago. The patient says his brother has since developed similar symptoms, and he has been told that it is caused by a virus, but he cannot remember its name. On examination, there is bilateral leg weakness with associated spasticity. Reflexes are intact, but vibration sense and proprioception are lost bilaterally. What is the most appropriate initial management of this patient’s condition?
1. Chemotherapy
2. Intravenous immunoglobulin
3. Symptomatic control only
4. Spinal surgery

A

3. Symptomatic control only

  • Human T cell lymphotropic virus 1 (HTLV-1) infection causes HTLV-1 associated myelopathy/tropical spastic paraparesis (HAM/TSP).
  • HAM/TSP usually presents with lower limb weakness and spasticity.
  • There is a high risk of developing HAM/TSP following solid organ transplant from an infected donor.
  • Though steroids and interferon have been trialed and are used, there is very limited evidence to support their effectiveness. Symptomatic treatment is the first-line.
83
Q

A 54-year-old African American male presents with progressive ataxia and left hemiparesis. He has a past medical history significant for focal segmental glomerulonephritis. Family history is notable for sarcoidosis in his mother as well as breast cancer in his sister. MRI brain shows a solitary homogenously enhancing lesion within the right internal capsule that is hyperintense to gray matter on T2 with restricted diffusion on diffusion-weighted imaging. Brain biopsy reveals a diffuse large B cell lymphoma. Which of the following is the next best step in management?
1. Corticosteroid administration
2. Systemic staging
3. Chemotherapy
4. Whole-brain radiotherapy

A

2. Systemic staging

  • Systemic staging is warranted prior to treatment because 4% to 12% of patients thought to have primary central nervous system lymphoma are found with systemic disease.
  • Primary central nervous system lymphoma comprises 5% of all primary brain tumors and 1% of non-Hodgkin lymphomas. The most common subtype is the diffuse large B cell variant.
  • Magnetic resonance imaging often shows hypointense lesions on T1, that are hyperintense on T2 with restricted diffusion on diffusion-weighted imaging. Post-contrast enhancement is typical. Up to 60% of lesions are solitary in immunocompetent individuals, while 30% to 80% of immunosuppressed patients present with multiple lesions.
  • Males are more frequently affected than females in immunocompetent individuals as well as in patients with AIDS. Lesions have a predilection for the periventricular white matter, as well as the frontal lobe and basal ganglia.
84
Q

What is one potential disadvantage of stereotactic radiosurgery for cerebral arteriovenous malformations?
1. Long recovery time
2. Lack of efficacy in small lesions
3. Required monitoring in the intensive care unit postoperatively
4. Latency period prior to obliteration of the arteriovenous malformation

A

4. Latency period prior to obliteration of the arteriovenous malformation

  • Stereotactic radiosurgery can be considered for arteriovenous malformations (AVMs) with a nidus smaller than 3 cm and often is considered for deeply located AVMs.
  • Potential advantages of radiosurgery include the ability to be performed as an outpatient, non-invasiveness, and lack of recovery period.
  • There is a latency period of 1 to 3 years between the time that stereotactic radiosurgery is performed and the AVM is actually obliterated.
  • During the latency period after stereotactic radiosurgery, the risk of hemorrhage from an AVM persists.
85
Q

A 38-year-old man presents to the emergency department after a workplace accident where a large number of industrial crates struck the patient. He is noted to have fractures to his right lower extremity but has no sensation below the waist. Radiographs reveal the superior articular processes of L1 to be posterior to the inferior articular processes of T12. The patient has no sensation in either of his lower extremities. He has poor rectal tone, and no motor function in either lower extremity. What is the next step in surgical management?
1. Emergent spinal decompression only at T12 and L1 within 8 hours
2. Urgent spinal decompression and fusion of his thoracolumbar spine within 24-48 hours
3. IV steroid injection and rapid cooling to protect spinal cord function
4. Observation only

A

2. Urgent spinal decompression and fusion of his thoracolumbar spine within 24-48 hours

  • Traumatic Lumbar Locked Facets are rare injuries that occur most commonly with high energy mechanisms in the workplace or in motor vehicle collisions in men aged 35-55.
  • Locked Facet Syndrome requires spinal decompression and fusion.
  • In the setting of a complete spinal cord injury, decompression should occur in the first 24-48 hours. However, there is no need for emergent decompression in a complete spinal cord injury.
  • Fusion is necessary after lumbar spinal decompression. IV Steroid use has not shown substantial benefit in spinal cord injuries and leads to more complications. Observation is not appropriate in the setting of Traumatic Locked Facets.
86
Q

A 4-year-old boy is brought to the hospital with a history of fall from the bed following a fight with his older sibling 3 hours back. His grandmother who witnessed the incident states that the boy was crying inconsolably for the initial 30 minutes, following which he has been fairly playful. Nevertheless, she is very scared if he has sustained any major injury. Physical examination reveals some pain during the extremes of neck rotations. Plain radiographs are suggestive of subluxation at the C2-C3 level. Which of the following radiological features is most likely to help identify the problem?
1. The intactness of the anterior vertebral body line
2. The intactness of the posterior vertebral body line
3. The intactness of spinolaminar line
4. Atanto-dens interval (ADI)

A

3. The intactness of spinolaminar line

  • The large head size of small children can cause a typical picture of pseudosubluxation at the C2-3 level on plain lateral radiographs, which may be mistaken for an underlying injury. Lateral radiographs in young children may be obtained by placing the children on a spine board with a recess for the head, so as to negate this disproportion between the child’s head and body.
  • Scischuk observed C2-3 pseudosubluxation as a normal variant on lateral cervical radiographs. He defined the posterior cervical (C1-3 spinolaminar) line as a reference to distinguish it from Hangman’s fracture.
  • Harrison observed this pseudo-subluxation phenomenon even in children aged between 14 and 18 years. In their series involving pediatric polytrauma patients, 21.7% presented with pseudo-subluxation.
  • In these scenarios, it may be difficult to differentiate between true and pseudo-subluxations. In physiological subluxation, Swischuk (C1-3 spinolaminar) line passes through or 1 mm anterior to the C2 posterior arch cortex. If the C2 posterior arch cortex lies within 1.5 mm posterior to Swischuk line, there is a suspicion for a true underlying injury, while a displacement greater than 2 mm indicates a true dislocation.
87
Q

A 66-year-old patient with angina, as well as claudication, reports feeling light-headed on exertion, especially when lifting and working with his arms. Duplex ultrasound showed subclavian artery peak systolic velocity of more than 240cm/second. The conventional cerebral angiography showed permanent retrograde vertebral flow. What is the severity grading of the characteristic clinical entity observed in the patient?
1. Grade 1
2. Grade 2
3. Grade 3
4. Grade 4

A

3. Grade 3

  • Subclavian steal syndrome results from occlusion of the subclavian artery secondary to atherosclerotic occlusion proximal to the origin of the vertebral artery. On the right side, the disease occurs in the innominate artery.
  • The severity of the subclavian steal phenomenon is divided into three grades.
  • Permanent retrograde vertebral flow signifies grade three severity of the disease process.
  • Grade one includes the reduced antegrade vertebral flow whereas grade two is characterized by the alternating flow - antegrade flow in the diastolic phase and retrograde flow in the systolic phase.
88
Q

A 65-year-old male patient complains of occipital headache, repeated vomiting, ataxia, and dizziness for the past 24 hours. The patient has a history of hypertension, diabetes, and smoking. Vital signs are blood pressure 150/80 mm Hg, pulse rate 95/min regular, and the respiratory rate 18/min. On physical examination, there is right lateral gaze palsy, with the remainder of the examination being normal. No hemiparesis is noted. Non- contrast CT scan is ordered, which shows a 3.5 cm hyperdense area in the cerebellum. What is the treatment of choice in such a patient?
1. External drainage alone without posterior fossa decompression
2. Surgical removal of hemorrhage with cerebellar decompression
3. Conservative therapy for cerebellar hemorrhage
4. Open craniotomy

A

2. Surgical removal of hemorrhage with cerebellar decompression

  • Current guidelines recommend surgical evacuation of cerebellar hemorrhages that are more than 3 cm in diameter. It is supported by the meta-analysis performed from observational studies.
  • The surgical evacuation was associated with improved survival outcomes with similar rates of functional outcome.
  • External drainage without posterior fossa decompression may lead to upward herniation of the cerebellar mass and is not the treatment of choice.
  • Although it lacks randomized control trials, based on the observational studies it has shown a reduction in mortality rate
89
Q

Each of the following patients has sustained a vertebral compression fracture. Identify the best candidate for kyphoplasty.
1. An 80-year-old male with an acute vertebral compression fracture and minimal back pain
2. A 72-year-old female on warfarin with a subacute vertebral compression fracture and 8/10 back pain refractory to pain medication and back bracing
3. A 66-year-old hospitalized female with 3/4 Society of Interventional Radiology (SIR) criteria and a painful, acute vertebral compression fracture refractory to conservative treatment
4. A 77-year-old female with a severely decreased bone mineral density seeking to prevent a potential osteoporotic vertebral compression fracture in the future

A

2. A 72-year-old female on warfarin with a subacute vertebral compression fracture and 8/10 back pain refractory to pain medication and back bracing

  • Vertebroplasty and kyphoplasty are indicated for acute, symptomatic compression fractures that are refractory to conservative management.
  • An asymptomatic vertebral compression fracture is an absolute contraindication to vertebroplasty and kyphoplasty.
  • Systemic and local infections such as osteomyelitis and discitis should be treated appropriately before attempting the procedure.
  • There are risks associated with vertebroplasty and kyphoplasty. Conservative management, including pain medication and bracing, should be considered as the first-line option in treating vertebral compression fractures.
90
Q

A 47-year-old woman presents to the clinic with a gradually worsening headache for the last 2 months. Brain MRI reveals a 55 x 45 mm well-circumscribed, extra-axial, dural-based mass at the left frontal parafalcine region. The mass is iso-intense to grey matter in both the T1- and T2-weighted images and shows intense and homogeneous enhancement following gadolinium administration. Sunburst appearance of vessels is evident on both the pre-contrast and post-contrast images. The mass displays restriction of diffusion on diffusion-weighted images (DWI) and apparent diffusion coefficient (ADC) mapping. Head CT reveals remodeling of the frontal bone adjacent to the mass. Which of the following is the most appropriate indication for a selective angiography in this case?
1. Verification of the diagnosis
2. Investigation of a possible accompanying vascular malformation
3. Evaluation for suspected sarcomatous degeneration
4. Preoperative embolization

A

4. Preoperative embolization

  • Meningiomas are the most common extra-axial brain tumors that present with a slow-growing dural-based mass. They are usually isointense to grey matter in both the T1- and T2- weighted images and show vivid, intense and homogeneous enhancement following gadolinium administration.
  • Meningiomas are highly vascular neoplasms that characteristically show sunburst appearance of vessels in both the pre-contrast and post-contrast MRI.
  • To reduce intraoperative blood loss, facilitate tumor resection and shorten the operation time, preoperative embolization can be applied to meningiomas.
  • Angiography for diagnostic purposes is not indicated in meningiomas.
91
Q

A 76-year-old man presents in the emergency department with visual complaints of a bilateral loss of vision in addition to a contraction in his visual field. After computerized tomography scanning, which shows a solid intracranial mass, he is sent for a consult at the ophthalmology department. Which location of the intracranial mass could give rise to these symptoms?
1. Olfactory groove, sphenoid wing, frontal lobe
2. Cerebellum, transverse fissure
3. Thalamus, third ventricle
4. Medulla oblongata, pons

A

1. Olfactory groove, sphenoid wing, frontal lobe

  • The reported involvement of the vision system points to the compressive effects of a tumor on the visual cortex or the retrobulbar tissues. Even though a tumor occurring elsewhere may have similar effects, it is understood that the closer that tumor is to the visual cortex or retrobulbar space, the more likely the patient will suffer damage to his vision.
  • The medulla is the farthest from the visual cortex/retrobulbar space among all the options and, as such, is least likely to cause this patient’s complaints.
  • Bilateral vision loss resulting from compression from a tumor in the retrobulbar space suggests a differential diagnosis of Foster Kennedy syndrome.
  • The olfactory groove, sphenoid wing, and frontal lobe all lie on or around the orbit, thus can be the cause of visual loss.
92
Q

A 25-year-old patient presented with a sudden onset worst headache of her life. An acute subarachnoid hemorrhage in the suprasellar cisterns was seen on a plain computed tomogram (CT) scan of her head. She is noted to have persistent hyponatremia. Based on her clinical presentation, what is the most likely site of the ruptured aneurysm?
1. Middle cerebral artery aneurysm
2. Posterior communicating artery aneurysm
3. Anterior communicating artery aneurysm
4. Basilar tip aneurysm

A

3. Anterior communicating artery aneurysm

  • Anterior communicating artery aneurysmal rupture may lead to bleeding into the hypothalamic-pituitary axis (HPA) territory.
  • Acute hematoma in this region leads to irritation and thereby activating the HPA axis.
  • The stimulation of the posterior pituitary leads to the excessive release of antidiuretic hormone (ADH), thereby leading to dilutional hyponatremia.
  • An aneurysmal bleed can lead to hyponatremia following the onset of cerebral salt wasting syndrome (CSW).
93
Q

A 35-year-old male is rushed to the emergency department following a motor vehicle collision. His presenting Glasgow Coma Scale score is 9. Urgent CT brain shows diffuse cerebral edema with obliteration of all cisterns. Which of the following is the most appropriate approach in the initial management of this patient’s traumatic brain injury?
1. Methylprednisolone
2. Hypertonic saline
3. Furosemide
4. Mannitol

A

2. Hypertonic saline

  • Hypertonic saline is the mainstay of medically managing traumatic intracranial hypertension.
  • It has been shown to effectively reduce intracranial pressure and improve cerebral and systemic perfusion.
  • It is safe, has a fast onset, sustained, and protracted action with minimal morbidities.
  • Mannitol has a risk of acute renal failure and rebound intracranial hypertension.
94
Q

A 17-year-old male presents with headaches and double vision. He had an upper respiratory tract infection 2 weeks ago. For the past week, he complains of headaches, graded 5/10, more prominent in the mornings, usually accompanied by nausea. His headache increased in severity to 8/10, and he now complains of double vision. He has no known medical comorbidities. On examination, his vital signs are within normal limits. Skin inspection shows petechial rash on the upper chest and extremities. Cranial nerve examination reveals papilledema bilaterally, as well as bilateral lateral rectus palsy. There are no motor or sensory deficits. There is positive nuchal rigidity and bilateral Babinski. A lumbar puncture reveals elevated opening pressure, WBC 700 cells/mm3, glucose 35 mg/dL, protein 85 mg/dL. Which is the most likely etiologic agent?
1. Escherichia coli
2. Neisseria meningitides
3. Streptococcus pneumoniae
4. Staphylococcus aureus

A

2. Neisseria meningitides

  • Neisseria meningitides are the most common etiology for bacterial meningitis in children, teens, and young adults.
  • Fever, neck stiffness, and altered mental status are the classic triad of symptoms for meningitis; however, all three are only present in 41% of cases of bacterial meningitis.
  • Bacterial meningitis typically results in low glucose and high protein levels in the cerebrospinal fluid. A neutrophil predominance on cell count would be expected.
  • The diagnosis would be confirmed with bacteria identified on gram stain or culture.
95
Q

A 60-year-old male presents to the clinic for worsening tremor. The patient has been diagnosed with essential tremor. The patient has remained compliant with his medication, however he denies any resolution of symptoms. After exhausting a multitude of pharmacologic options, the clinician offers deep brain stimulation (DBS). What structure should be targeted when performing DBS to treat essential tremor?
1. Ventral intermediate thalamus
2. Corpus Collosum
3. Nucleus tractus solitarius
4. Lamina terminalis

A

1. Ventral intermediate thalamus

  • Both essential tremor (ET) and parkinsonian tremor respond to deep brain stimulation.
  • The probe for essential tremor should be placed in the ventral intermediate thalamus.
  • Medical treatment includes propranolol, ethyl alcohol, and primidone, however, in the setting of treatment-resistance, DBS should be considered.
  • DBS is approved by the US Food and Drug Administration (FDA) for the treatment of essential tremor, dystonia, Parkinson’s disease, and treatment-refractory obsessive- compulsive disorder (OCD).
96
Q

Histopathological analysis of a specimen of an intraventricular lesion shows cells increased in density and arranged in papillary fronds. Nuclear pleomorphism and necrosis are absent. What is the most likely diagnosis?
1. Epidermoid
2. Choroid plexus carcinoma
3. Choroid plexus papilloma
4. Ependymoma

A

3. Choroid plexus papilloma

  • Choroid plexus tumors retain the normal architecture of choroid plexus, i.e., frond-like arrangement. However, the cell density will be increased.
  • They do not show nuclear pleomorphism or necrosis on microscopy.
  • They show less than 2 mitoses per 10 high power fields.
  • Choroid plexus carcinomas have more mitoses, show nuclear pleomorphism, and necrosis.
97
Q

A 17-year-old female is being evaluated for upward gaze palsy. A computed tomography (CT) of the head revealed the presence of a pineal region tumor. The endoscopic third ventriculostomy (ETV) guided biopsy revealed large, epithelioid cells with abundant periodic acid–Schiff positive (PAS+) cytoplasm mixed with mature lymphocytes. Cerebrospinal fluid cytology is negative. Which of the following is the best treatment strategy for this patient?
1. Surgical resection
2. Craniospinal irradiation
3. Localized radiotherapy
4. Chemotherapy combined with ventricular radiotherapy

A

4. Chemotherapy combined with ventricular radiotherapy

  • The presence of large, epithelioid cells with abundant PAS+ cytoplasm mixed with lymphocytes is characteristic of a germinoma. Germinoma accounts for more than 50% of lesions in the pineal region.
  • Germinomas can be localized or disseminated. Localized pure germinoma is best treated with chemotherapy combined with reduced dose ventricular radiotherapy. Craniospinal irradiation is no longer recommended for localized disease.
  • Germinomas typically have a very good response to radiation therapy and chemotherapy. They are prognostically better than non-germinomatous lesions.
  • Surgery is not indicated for germinoma. Localized radiotherapy is associated with a higher risk of relapse.
98
Q

Once the physician confirms that the needle tip is in the pterygopalatine fossa, stimulation of which of the nerves confirm the correct placement of the radiofrequency probe before attempting thermocoagulation?
1. Maxillary branches
2. Greater palatine nerves
3. Lesser palatine nerves
4. Posterior lateral nasal nerves

A

4. Posterior lateral nasal nerves

  • Test stimulation of maxillary branches is confirmed by paraesthesia of upper teeth and gums. That suggests the tip is superolateral to the target. The physician should redirect the needle caudal and medially.
  • Test stimulation of greater palatine nerves is confirmed by paraesthesia of hard palate. That suggests the tip is anterior, lateral, and caudal to the target. The physician should redirect the needle caudal and posteromedially.
  • Test stimulation of lesser palatine nerves is confirmed by paraesthesia of hard palate. That suggests the tip is anterior, lateral, and caudal to the target. The physician should redirect the needle caudal and posteromedially.
  • Test stimulation of posterior lateral nasal nerves is confirmed by paraesthesia of root of the nose. This is the correct needle placement. Once this target is achieved, radiofrequency ablation should be attempted.
99
Q

An 82-year-old male patient reported hearing difficulty and speech recognition problems over the last year. The patient attributed these symptoms to aging and did not seek medical attention. He started experiencing a feeling of spinning of his surroundings 2 months ago. This led to a non-contrast CT scan which revealed an underlying right-sided cerebellopontine mass. MRI further solidified the CT findings. Audiometric examination revealed a severe sensorineural hearing loss in the right ear. His comorbid illnesses include heart failure, hypertension, diabetes mellitus, chronic kidney disease, and chronic obstructive pulmonary disease. Family history includes congestive heart failure in both parents. He currently lives in a nursing home. What is the most appropriate treatment option for this patient?
1. Observation
2. Radiotherapy
3. Surgical resection via the retrosigmoid approach
4. Surgical resection translabyrinthine approach

A

2. Radiotherapy

  • Radiotherapy is suitable for elderly patients or poor surgical candidates. Tumor growth is halted successfully. However, worsening of hearing deficits and cranial neuropathies are common.
  • Tumor recurrence can also occur and serial follow-up imaging should be taken routinely. Radiation-induced-tumour is also a concern in younger patients.
  • Microsurgery offers a definitive solution. The goal of microsurgery is complete tumor removal as well as hearing preservation. The different surgical approaches that can be used are translabrynthine, middle fossa and retrosigmoid approach.
  • The most commonly used approach is retrosigmoid (RS) or the sub-occipital approach. This approach allows hearing preservation while providing adequate exposure.
100
Q

A 55-year-old patient was operated with laminectomy and medial facetectomy at L4-5 level for disc herniation, and hypertrophic facet joints with focal canal stenosis. Post-operatively, the patient complaint of persistent pain at the operative site, especially while ambulating. The patient has no fever or wound soakage. The erythrocyte sedimentation rate (ESR) and the C-reactive protein (CRP) levels are within the normal range. What is the next step in the management of this patient?
1. Escalate the analgesic dosage
2. Antibiotics
3. MRI spine
4. Dynamic X-ray of the lumbosacral spine

A

4. Dynamic X-ray of the lumbosacral spine

  • Over decompression of the spine can weaken the facet joints and the par interarticularis thereby leading to instability.
  • Joint instability leads to characteristic localized pain, especially during ambulation, due to the irritation of the nerve of Luschka.
  • The mainstay of diagnosing joint instability is by performing dynamic X-ray images in flexion and extension.
  • The probability of discitis with a normal erythrocyte sedimentation rate and C-reactive protein values is minimal. There is no sciatica in the patients, so the chance of the recurrent disc at the same level is also low.